Vous êtes sur la page 1sur 58

EXAMEN 2008 1. Cul de los siguientes pacientes tiene una indicacin ms clara de tratamiento quirrgico de enfermedad por reflujo?

: 1. Paciente de 80 aos con hernia hiatal de mediano tamao y pirosis frecuente que responde bien a 20 mg/da de omeprazol. 2. Paciente de 56 aos con molestias epigstricas tipo flatulencia y pirosis intermitente que responde slo ligeramente al tratamiento con omeprazol. 3. Paciente de 27 aos con sndrome depresivo y molestias retroesternales que no alivian en absoluto con el tratamiento con inhibidores de la bomba de protones. 4. Paciente de 58 aos con pirosis diaria diurna y nocturna de ms de 10 aos de evolucin y que permanece asintomtico en los perodos que es tratado con dosis de omeprazol de 40 mg/da o superiores y cuyos sntomas recidivan inmediatamente al reducir esta dosis. 5. Paciente de 66 aos de edad con esofagitis erosiva y antecedentes de infarto de miocardio hace 1 ao con insuficiencia cardiaca residual. 2. Cul de las siguientes afirmaciones es correcta en relacin al divertculo faringoesofgico o divertculo de Zenker?: 1. 2. 3. 4. 5. Se trata de un divertculo por traccin. Suele diagnosticarse en pacientes jvenes. El tratamiento incluye la miotoma del msculo cricofarngeo. La pirosis es su sntoma principal. Se localiza siempre en la cara anterior de la hipofaringe.

3. La gastroscopia es una prueba diagnstica fundamental para el estudio del paciente con hemorragia digestiva, y la urgencia en la realizacin de la misma depender de la magnitud de la hemorragia digestiva. Podra sealar cul de las siguientes situaciones NO expresa una mayor magnitud y por tanto gravedad de la hemorragia?: 1. Hipotensin y taquicardia. 2. Disminucin del hematocrito y hemoglobina, 3. Pacientes que requieren transfusin para mantener la estabilidad hemodinmica. 4. Lavado gstrico por sonda nasogstrica con abundante sangre roja, que no se aclara tras lavados repetidos con abundante volumen. 5. Repetidos y frecuentes episodios de hematemesis con sangre roja y melenas. 4. Cul de las siguientes pautas es ms probable que sea efectiva en la erradicacin de Helicobacter Pylori despus del fracaso de un primer tratamiento con Omeprazol (20 mg/12h), Amoxicilina (lg/12h) y Claritromicina (500 mg/12h) durante siete das?: 1. Repetir nuevamente la pauta inicial pero mantenida durante 14 das. 2. Repetir la pauta inicial pero cambiando Amoxicilina por Tetraciclina (500 mg/6h). 3. Repetir la pauta inicial pero cambiando Claritromicina por Metronidazol (500 mg/8h). 4. Es necesario practicar cultivo de biopsia gstrica y antibiograma para decidir la combinacin antibitica con mayores garantas de xito. 5. Administrar Omeprazol (20 mg/12h), Amoxicilina (1 g/12h) y Levofloxacino (500 mg/12h) durante 10 das. 5. Un paciente de 46 aos de edad acude a urgencias por vmitos en posos de caf y melena. El hematocrito de entrada es del 33%, su presin arterial es de 110/70 mmHg y su frecuencia cardiaca de 87 latidos por minuto. Un estudio endoscpico practicado 5 horas despus de su ingreso revela alguna erosin superficial limpia en el tercio distal del esfago y varias erosiones superficiales en la zona antral prepilrica, una de ellas con un punto de hematina en su base. No quedan restos hemticos en la cavidad gstrica. El paciente se haba medicado con diclofenaco 150 mg/da los ltimos 8 das por una ciatalgia. Cul sera la actitud ms recomen dable?: 1. Tratamiento con inhibidores de la bomba de protones por va oral y alta hospitalaria. 2. Tratamiento erradicador de Helicobacter pylori de forma emprica y alta hospitalaria. 3. Tratamiento hemosttico con sonda de calor de la erosin con hematina en su base y perfusin endovenosa de inhibidores de la bomba de protones durante tres das.

4. Ayuno total y tratamiento con perfusin endovenosa de inhibidores de la bomba de protones durante 48 horas. 5. Colocacin endoscpica de un hemoclip en la lesin erosiva con signos de hemostasia reciente. 6. Una mujer de 45 aos, madre de un nio celaco y sin sntomas ni signos de patologa digestiva, es evaluada por anemia ferropnica detectada en analtica rutinaria. Cul de los siguientes enunciados es cierto respecto a este caso?: 1. l riesgo de esta mujer de padecer enfermedad celaca es similar al de la poblacin general. 2. La ausencia de diarrea hace que se pueda excluir la enfermedad celaca como causa de la anemia de esta paciente. 3. La determinacin del alelo HLA-DQ2, de resultar positiva, establecera el diagnstico de enfermedad celaca. 4. La evaluacin inicial de esta paciente debera incluir la determinacin de anticuerpos antiendomisio y/o anticuerpos antitransglutaminasa tisular. 5. La edad de la paciente permite excluir la enfermedad celaca del diagnstico diferencial. 7. Respecto a la enfermedad de Crohn. Cul de los siguientes enunciados es cierto?: 1. El proceso inflamatorio est limitado a la mucosa y a la submucosa superficial, y las capas ms profundas permanecen respetadas. 2. Para establecer el diagnstico de enfermedad de Crohn se precisa la presencia de granulomas no caseificantes. 3. El diagnstico de enfermedad de Crohn no debe considerarse en individuos mayores de 50 aos. 4. El tabaco es un factor protector para la enfermedad de Crohn. 5. No todos los pacientes con fstulas perianales presentan signos endoscpicos de inflamacin en el coln. 8. A un paciente de 60 aos, sin antecedentes familiares de cncer colorrectal o de poliposis, se le halla en una colonoscopia, un nico plipo ssil de 0,5 cm en el sigma que se reseca por completo en un solo fragmento en el mismo acto endoscpico. La colonoscopia ha sido completa hasta el ciego y la preparacin del coln era excelente. La anatoma patolgica del plipo revela un adenoma tubular con displasia de bajo grado. Cul de las siguientes recomendaciones de seguimiento sera la ms adecuada?: 1. Programar un nuevo control colonoscpico dentro de 1 ao. 2. Programar un nuevo control colonoscpico dentro de 5 aos. 3. No hace falta programar una nueva colonoscopia porque el plipo ya ha sido resecado y no tena displasia de alto grado. 4. Para determinar con mayor precisin el intervalo de tiempo en el cual hay que repetir una nueva colonoscopia es preciso realizar un anlisis de inestabilidad de microsatlites en el plipo resecado y eventualmente secuenciacin de los genes reparadores del ADN. 5. Repetir un nuevo control colonoscpico a los tres meses con biopsias de la cicatriz de la base de reseccin del plipo. 9. Un hombre de 28 aos asintomtico es remitido a consultas por detectarse en analtica del examen de rutina laboral una cifras de bilirrubina total en sangre de 3,8 mg/100 ml, con resto del perfil sanguneo heptico normal. Refiere cifras ocasionalmente similares de bilirrubina en exmenes previos. Qu prueba cree importante para orientar el diagnstico?: 1. Colangiopancreatografa endoscpica retrgrada (CPRE) para estudiar la va biliar. 2. No es preciso hacer ms exploraciones. 3. Determinacin de bilirrubina total y fraccionada y frotis sanguneo. 4. Colecistografa oral, 5. Determinacin de virus hepatotropos. 10. Cul de las siguientes afirmaciones relativas al tratamiento de la hepatitis crnica viral es FALSA?: 1. Una respuesta viral mantenida (negativizacin del ARN) en la hepatitis crnica C se considera curacin de la infeccin viral. 2. Una respuesta viral mantenida (negativizacin del ADN) en la hepatitis crnica B se considera curacin de la infeccin viral.

3. La ausencia de respuesta viral temprana permite interrumpir el tratamiento en pacientes con hepatitis crnica C, evitando efectos secundarios y costes econmicos innecesarios. 4. La duracin del tratamiento antiviral en la hepatitis crnica C depende del genotipo viral. 5. El tratamiento antiviral indicado en la hepatitis crnica por virus C es la asociacin de interfern alfa pegilado y ribavirina. 11. Hombre de 60 aos, diagnosticado de cirrosis heptica virus C, en situacin clnica Child B, que presenta hemorragia por varices gstricas. El tratamiento mdico, la terapia endoscpica y el taponamiento han fracasado, continuando con hemorragia digestiva por varices. El tratamiento ms adecuado ser: 1. Trasplante heptico urgente. 2. Realizacin de una ciruga resectiva: desconexin cigo-portal {tcnica de Sugiura). 3. Realizacin de una portocava calibrada (shunt portosistmico quirrgico). 4. Shunt transyugular porto sistmico intraheptico. 5. La mejor ciruga derivativa de urgencia es la derivacin selectiva espleno-renal distal de Warren. 12. Un hombre de 52 aos acude a consulta refiriendo un cuadro de cansancio generalizado, leve prdida de peso, dolores articulares en muecas y rodillas y disminucin de la libido. No refiere hbitos txicos ni ingesta de medicamentos. A la exploracin llama la atencin un bronceado de la piel (no relacionado con toma de sol) y discreta hepatomegalia. En la analtica realizada destaca: glucosa basal 180 mg/dl; GOT: 78 U/1; GPT: 89 U/1; ferritina 650 ng/ml; transferrina: 260 mg/dl; saturacin de transferrina: 66^6%; hierro srico: 220 ug/dl. Qu otra prueba complementaria entre las siguientes solicitara inicialmente para llegar al diagnstico?: 1. 2. 3. 4. 5. Prueba de sobrecarga oral a la glucosa. Estudio gentico de la mutacin C282Y, Carga viral del virus de la hepatitis C. Niveles de testosterona en sangre. Resonancia magntica de las articulaciones de la mueca.

13. Ante la presencia de una masa heptica asintomtica descubierta de forma casual, es FALSO: 1. El diagnstico diferencial entre masa slida y qustica se realizar mediante ecografa. 2. Se debe establecer el diagnstico diferencial entre hiperplasia nodular focal y el adenoma heptico. 3. El hemangioma cavernoso se diagnostica por resonancia con contraste de gadolinio al ser muy intenso en T2. 4. El tratamiento de eleccin de la hiperplasia nodular focal es quirrgico por la posibilidad de malignidad. 5. La biopsia heptica de la lesin ser de utilidad para instaurar una terapia no quirrgica. 14. Una de las siguientes patologas NO es un factor predisponerte para el desarrollo de un colangiocarcinoma: 1. 2. 3. 4. 5. Litiasis intraheptica. Parasitosis por Clonorchis Sinensis. Colangitis esclerosante primaria. Hemocromatosis. Enfermedad de Caroli.

15. Cundo sospecharemos que un paciente diagnosticado previamente de pancreatitis crnica est desarrollando una insuficiencia pancretica exocrina grave?: 1. 2. 3. 4. 5. El paciente explica prdida de apetito. Se le detecta hiperglicemia en varias analticas realizadas. El paciente refiere deposiciones amarillentas y brillantes con prdida de peso. Su enfermedad ya lleva muchos aos de evolucin. El paciente se queja de dolor incapacitante.

16. En un paciente con pancreatitis aguda que presenta mal estado general, leucocitosis de 20.000/uX, fiebre de 39C, insuficiencia renal, disnea y shock, la actuacin recomendada es: 1. 2. 3. 4. 5. Laparotoma de urgencia. Nutricin parenteral total. Albmina i.v. hasta que la albmina srica supere los 3 g/L. Dextrano 60. Puncin del pncreas con aguja fina y cultivo.

17. Un paciente de 25 aos inici hace 6 semanas un cuadro de diarrea con sangre y fiebre por lo que fue tratado con amoxicilina/clavulnico durante 7 das sin mejora. Ante la persistencia de los sntomas se practica una colonoscopia que demuestra un recto normal y la presencia de aftas en sigma y lceras longitudinales en colon transverso. Cul es el diagnstico de sospecha ms probable?: 1. 2. 3. 4. 5. Colitis por Clostridium Difficile. Colitis ulcerosa, Colitis amebiana. Enfermedad de Crohn. Colitis isqumica.

18. Un nio de 10 aos sufri un accidente de bici cleta y hubo que intervenirlo quirrgicamente por un hemoperitoneo. Exista una ruptura del bazo y hubo que practicar una esplenectoma urgente. Todas las entidades que se refieren a continuacin son ms frecuentes en este nio que en la poblacin normal, SALVO una. Seale sta: 1. 2. 3. 4. 5. Bacteriemias. Ulcus duodenal. Trombosis profunda. Neumonas. Meningitis graves.

19. Qu tenemos que hacer cuando en una pancreatitis aguda grave se demuestra por el cultivo, realizado en la puncin-aspiracin con aguja fina guiada por TAC abdominal, infeccin de la necrosis pancretica?: 1. 2. 3. 4. 5. Iniciar tratamiento antibitico y mantener actitud conservadora. Cambiar la nutricin parenteral por enteral. Tratamiento quirrgico: desbridamiento o necrosectoma. Iniciar tratamiento con un inhibidor de las proteasas. Aumentar el tratamiento analgsico.

20. Un paciente acude a su consulta y usted hace el diagnstico de presuncin de diverticulitis aguda, porque relata cuatro signos frecuentes de esta enfermedad. Uno de los siguientes NO suele acompaar a la diverticulitis aguda de sigma: 1. Dolor en fosa ilaca izquierda. 2. Escalofros. 3. Fiebre. 4. Cambio en el ritmo intestinal, 5. Rectorragias. 21. Gimbernat dijo en el siglo XIX que las enfermedades que ms comnmente afligen a la humanidad son sin duda las hernias. Hasta finales del siglo XX stas eran la causa ms frecuente de oclusin intestinal. En las ltimas dcadas la causa ms frecuente de la obstruccin intestinal mecnica es: 1. Siguen siendo las hernias, suponiendo un 60% de las causas de leo. 2. Las tumoraciones malignas, que suponen un 60% de las causas de leo. 3. El sndrome adherencia! postoperatorio, que supone el 60% de las causas de leo. 4. Los vlvulos e invaginaciones, que sumados en nios y adultos suponen un 60% de las causas de leo.

5. Los sndromes metablicos tipo diabetes y otros, lo que sumado a la pluripatologa y ancianidad de los pacientes quirrgicos actuales suponen un 60% de las causas de leo. 22. Un hombre de 72 aos de edad con slo antecedentes de hipercolesterolemia bien controlada, leve depresin y enfermedad por reflujo gastroesofgico controlada con omeprazol, acude al Servicio de Urgencias por presentar desde hace 12 horas dolor intenso en fosa iliaca izquierda y aumento del nmero de deposiciones (hasta 4/da), lquidas pero sin productos patolgicos. A la exploracin se observa dolor a la palpacin superficial y profunda en fosa iliaca izquierda, ruidos presentes y no signos de irritacin peritoneal. La analtica urgente es normal salvo leucocitosis (16.300) con desviacin izquierda. Se le realiza un TAC abdominal urgente en el que se describe imgenes sugerentes de diverticulosis con signos de diverticulitis aguda sin datos de perforacin. Qu actitud teraputica considera ms correcta?: 1. Realizacin de colonoscopia urgente. 2. Colocacin urgente de un drenaje percutneo, dirigido por TAC, en rea inflamada. 3. Intervencin quirrgica urgente para extirpar zona del colon inflamada. 4. Alta domiciliaria, recomendando dieta rica en fibra y que acuda nuevamente a urgencias si empeora la situacin del paciente. 5. Observacin hospitalaria con dieta absoluta y administracin de ciprofloxacino y metronidazol por va i.v. 23. En cul de las siguientes situaciones clnicas, el pulso paradjico (disminucin de la presin arterial de ms de 10 mmHg durante la inspiracin), no est presente en la exploracin fsica?: 1. 2. 3. 4. 5. Estenosis artica. Taponamiento cardiaco. Pericarditis constrictiva. Embolia pulmonar. Enfisema pulmonar (cor pulmonale).

24. Est contraindicado usar betabloqueantes como tratamiento en sujetos hipertensos que tengan adems una de las siguientes circunstancias acompaantes, SALVO: 1. 2. 3. 4. 5. Bloqueo aurculo-ventricular de 2o grado. Asma bronquial, Enfermedad del ndulo sinusal. Diabetes mellitus en tratamiento con hipoglucemiantes. Insuficiencia cardiaca.

25. Hombre de 55 aos, fumador de 20 cigarrillos/da y bebedor de ms de 90 gramos de alcohol/da. Ingresa por presentar en los ltimos 2 meses disnea progresiva hasta hacerse de repo so, ortopnea y crisis de disnea paroxstica nocturna. En la exploracin destaca soplo pansistlico, crepitantes de gruesa burbuja diseminados y edemas en extremidades inferiores. Electro cardiograma: fibrilacin auricular con respuesta ventricular a 130 Ipm y bloqueo completo de rama izquierda. En la radiografa de trax pre sentaba cardiomegalia global, derrame pleural bilateral, edema intersticial en bases y lneas B de Kerley. Cul sera su primer diagnstico de sospecha?: 1. 2. 3. 4. 5. Miocardiopata restrictiva. Pericarditis crnica constrictiva. Miocardiopata dilatada. Miocardiopata hipertrfica obstructiva. Cor pulmonale crnico.

26. Cul de los siguientes hallazgos ecocardiogrficos es un criterio mayor en el diagnstico de la Endocarditis Infecciosa?: 1. 2. 3. 4. Derrame pericrdico. Prolapso de un festn de la vlvula mitra!. Insuficiencia mitral moderada. Absceso periartico.

5. Insuficiencia artica severa. 27. Un paciente de 63 aos, con Insuficiencia Cardiaca por cardiopata hipertensiva, en situacin estable (en clase funcional I segn grado de disnea), presenta en el Ecocardiograma, Disfuncin Sistlica (Fraccin de Eyeccin < 35%). Qu grupo de frmacos estara ms indicado como tratamiento inicial?: 1. 2. 3. 4. 5. Digitlicos. Inhibidores de la ECA. Antagonistas de Calcio. Betabloqueantes. Diurticos.

28. En relacin con el empleo de la trombolisis en el infarto agudo de miocardio, seale la afirmacin FALSA: 1. 2. 3. 4. 5. Es til para disminuir el tamao de la zona infartada. Es til para disminuir las arritmias. Es til para disminuir la mortalidad. Es til para limitar la disfuncin ventricular izquierda. Slo es til en las primeras horas post-infarto.

29. El mecanismo principal de accin de la tenecteplasa (TNK) en los sndromes coronarios agudos con elevacin del segmento ST es: 1. 2. 3. 4. 5. Estimular el complejo protrombinasa y la formacin de trombina. Estimular la conversin de plasmingeno en plasmina. Bloquear el receptor GP Hb/EIa de las plaquetas. Inhibir la formacin de trombina y plasmingeno. Estimular la produccin de heparinoides.

30. Paciente de 65 aos de edad, con antecedentes de haber sufrido un infarto de miocardio hace dos aos. Acude aj Hospital por un cuadro de malestar general, sudoracin y palpitaciones. En el electrocardiograma se observa un ritmo regular a 170 latidos por minuto con complejos QRS de 0.14 segundos. Cul sera su primer diagnstico?: 1. Fibrilacin auricular paroxstica. 2. Taquicardia nodal. 3. Taquicardia ventricular. 4. Taquicardia paroxstica supraventricular. 5. Flutter auricular 31. Una de las siguientes respuestas, referidas a la arritmia cardiaca, fibrilacin auricular, es FALSA: 1. Deben descartarse factores precipitantes como hipertiroidismo o embolismo pulmonar. 2. La anticoagulacin est indicada en todos los casos. 3. En situaciones de inestabilidad hemodinmica es adecuado el tratamiento de cardioversin elctrica. 4. Cuando el dimetro de la aurcula izquierda es mayor de 45 mm, puede ser difcil revertir a ritmo sinusal. 5. Cuando el dimetro de la aurcula izquierda es mayor de 45 mm puede ser difcil mantener el ritmo sinusal, si se consigue la reversin. 32. Se considera indicacin de desfibrilador implantable:

1. Extrasstoles ventriculares complejos sintomticos en paciente sin cardiopata. 2. Taquicardia ventricular incesante. 3. Fibrilacin ventricular que ocurre en la primera hora tras un infarto agudo de miocardio. 4. Parada cardiaca por fibrilacin ventricular en paciente con infarto de miocardio crnico anterior extenso. 5. Sncopes de repeticin en paciente con estudio cardiolgico normal.

33. En cul de estas posibles causas desencadenantes de Insuficiencia Cardiaca NO existe incremento del gasto cardiaco?: 1. 2. 3. 4. 5. 34. Tirotoxicosis. Embarazo. Anemia. Fiebre. Taquiarritmia. Seale la respuesta INCORRECTA respecto a la estenosis artica:

1. Suele requerir tratamiento quirrgico en personas mayores. 2. La edad no suele ser una contraindicacin para el recambio valvular. 3. La muerte sbita es una complicacin poco comn de los pacientes con estenosis artica sintomtica. 4. Cuando la estenosis artica se hace sintomtica est indicada la sustitucin valvular. 5. Los pacientes con estenosis artica que desarrollan angina tienen un elevado riesgo de mortalidad. 35. Cul de los siguientes diagnsticos es ms pro bable en un paciente de 33 aos con antecedentes familiares de muerte sbita que acude por dolor torcico intenso, de inicio brusco y en reposo y que en la exploracin presenta un soplo diastlico precoz y ausencia del pulso radial izquierdo?: 1. 2. 3. 4. 5. 36. 1. 2. 3. 4. 5. Embolia de pulmn. Derrame pericrdico severo. Neumotrax espontneo. Infarto de miocardio con embolia a la arteria radial izquierda. Diseccin artica. Cul es el tumor benigno cardiaco ms frecuente en adultos?: Rabdomioma. Lipoma. Fibroma. Mixoma. Hemangioma. Seale la respuesta correcta sobre la ciruga de la trombosis venosa:

37.

1. Cuando afecta al cayado de la safena interna puede ligarse ste por debajo del trombo y evitarse as la anticoagulacin. 2. La anticoagulacin es obligatoria cuando se practica la extirpacin del sistema superficial. 3. El tratamiento quirrgico de las varices de miembros inferiores requiere el ingreso hospitalario de los pacientes. 4. El edema y las lceras son complicaciones poco frecuentes de las varices. 5. No es bueno administrar antiinflamatorios tpicos en las varices superficiales. 38. La alteracin ms frecuente de las pruebas de funcin pulmonar en los supervivientes del Sndrome de Distrs Respiratorio del Adulto es: 1. 2. 3. 4. 5. 39. 1. 2. 3. 4. 5. Disminucin de la complianza pulmonar. Disminucin de la capacidad de difusin. Aumento de resistencia de la va area. Hipertensin pulmonar. Disminucin de la capacidad pulmonar total. La siguiente gasometra arterial: PH 7.40, Pa02 98, PaC02 38, HC03 25, refleja: Acidosis metablica. Alcalosis metablica. Valores normales. Alcalosis respiratoria. Acidosis respiratoria.

40. En el asma persistente leve. Cul de los siguientes esquemas teraputicos puede considerarse de eleccin?: 1. 2. 3. 4. 5. Salmeterol a demanda. Salbutamol pautado ms salbutamol a demanda. Budesonida a demanda ms salbutamol a demanda. Budesonida pautada ms salbutamol a demanda. Budesonida pautada ms salbutamol pautado.

41. Por qu mecanismo aparece la "hiperinsuflacin dinmica" durante el ejercicio en los enfermos con EPOC?: 1. Por alteraciones en la relacin "ventilacin/perfusin". 2. Por vaciado incompleto de unidades alveolares a causa del aumento de la frecuencia respiratoria. 3. Por vaciado incompleto de unidades alveolares a causa de una disminucin en la "orden respiratoria. 4. Por los problemas mecnicos asociados a una alcalosis metablica. 5. Por los problemas mecnicos asociados a una alcalosis respiratoria. 42. Un paciente de 60 aos, fumador, con historia de disnea de un ao de evolucin presenta una auscultacin de crepitantes y se comprueban acropaquias. La TC torcica demuestra una afectacin reticular en la periferia del pulmn. El diagnstico ms probable es: 1. Neumonitis por hipersensibilidad. 2. Neumona organizada criptognica. 3. Fibrosis pulmonar idioptica. 4. Neumona intersticial no especfica. 5. Neumona eosinfila crnica. 43. A un paciente obeso y roncador intenso, sin somnolencia diurna, con antecedentes de insuficiencia cardiaca, se le practica una polisomnografa despus de haber detectado su pareja pausas respiratorias durante el sueo. El registro muestra un ndice de apnea-hipopnea de 4/hora a expensas de apneas obstructivas, una Sa02 media durante el sueo de 94% y una Sa02 mnima de 6%. El diagnstico es: 1. Se trata de un roncador simple que no precisa tratamiento. 2. Se trata de un sndrome de apnea del sueo subsidiario de tratamiento con CPAP nasal. 3. Se trata de un sndrome del sueo subsidiario de dieta hipocalrica y dormir en decbito lateral. 4. Se trata de un sndrome de apneas centrales secundarias a la insuficiencia cardiaca. 5. Los resultados de polisomnografa no son compatibles con la historia clnica. 44. es: 1. 2. 3. 4. 5. El hallazgo ms frecuente en la radiografa de trax en un paciente con asma

Hiperinsuflacin pulmonar. Condensaciones alveolares bilaterales y difusas. Radiografa de trax normal. Engrosamiento de paredes bronquiales. Neumomediastino.

45. Un paciente de 43 aos acude a urgencias por disnea, tos y fiebre de una semana de evolucin y en la radiografa de trax presenta una cavidad de paredes lisas en lbulo superior derecho rodeada por reas de consolidacin alveolar. Cul es la conducta inicial ms correcta?: 1. El paciente puede ser dado de alta con tratamiento antibitico y realizar una radiografa de control en un mes. 2. Se debe realizar un estudio de tomografa computarizada (TC) torcica ante la sospecha de neoplasia pulmonar. 3. El paciente puede ser bacilfero y se debe realizar un anlisis del esputo. 4. Se debe obtener un diagnstico citolgico mediante broncoscopia o puncin percutnea.

5. Se debe completar el estudio con una Resonancia Magntica con gadolinio. 46. Cul sera la primera exploracin radiolgica que se hara en un paciente con dolor torcico?: 1. 2. 3. 4. 5. Rx PA y Lateral del trax en inspiracin. Rx PA y Lateral del trax en espiracin. Rx en decbito supino. Rx oblicuas de ambos pulmones. Rx en proyeccin lordtica.

47. Mujer de 55 aos, postmenopusica y fumadora activa, que consulta por dolor de hombro izquierdo irradiado a brazo de varias semanas de duracin. En la exploracin fsica destaca ptosis y miosis del ojo izquierdo. Cul sera su actitud?: 1. 2. 3. 4. 5. Solicitar estudio de osteoporosis. Solicitar estudio oftalmolgico. Realizar anticuerpos anti-receptor de acetilcolina. Solicitar TAC cerebral. Solicitar radiografa simple de trax.

48. Mujer de 70 aos que acude a urgencias por cuadro sincopal. Intervenida de neoplasia de coln hace 2 semanas. En la exploracin: presin venosa a 4 cm sobre horizontal, 102 latidos por minuto, 35 respiraciones por minuto. Temperatura: 37,8C, dolorimiento abdominal difuso, resto de exploracin sin datos patolgicos; pulsioximetra: saturacin de oxgeno basal: 86%. ECG: ritmo sinusal a 100 Ipm, inversin de ondas T de VI a V4. Radiografa de trax: elevacin de hemidiafragma izquierdo con pinzamiento senocostofrnico. Cul es el diagnstico ms probable?: 1. 2. 3. 4. 5. Infeccin respiratoria. Infarto de miocardio con Sndrome de Dressler. Taponamiento cardaco. Tromboembolismo pulmonar masivo. Dehiscencia de sutura con distress respiratorio.

49. Paciente de 28 aos con cuadro de dos das de evolucin de tos y fiebre de 39. En la radiografa de trax se observa la existencia de un derrame pleural que ocupa la mitad del hemitrax izquierdo. Los hallazgos bioqumicos de la toracocentesis son los siguientes: pH 7,10, LDH 1200 u/L, protenas 4,2, ADA 70. Cul ser la conducta ms apropiada a seguir?: 1. Iniciar tratamiento antibitico emprico y esperar los resultados del cultivo del lquido pleural para colocar un tubo de drenaje torcico. 2. Colocar un drenaje pleural, instilar fibrinolticos intrapleurales e iniciar tratamiento antibitico emprico. 3. Iniciar tratamiento antibitico emprico y repetir la toracentesis al cabo de 24 horas para valorar la actitud a seguir en funcin de la evolucin de los parmetros bioqumicos del lquido pleural. 4. Colocar un tubo de drenaje torcico e iniciar tratamiento antibitico emprico. 5. Efectuar una broncoscopia para toma de muestras microbiolgicas, colocar un tubo de drenaje torcico e iniciar tratamiento antibitico emprico. 50. Paciente de 62 aos, fumador importante, que presenta masa en lbulo inferior derecho con diagnstico de carcinoma no microctico de pulmn que infiltra la grasa mediastnica y rodea al esfago, desplazndolo. No hay evidencia de adenopatas mediastnicas afectadas por tumor. El paciente es operable, Cul de estas actitudes le parece ms adecuada llegado este momento?: 1. El paciente presenta un carcinoma broncognico no microctico estadio IV (T4N0M0), por lo que no es resecable. Se debe tratar con quimioterapia y radioterapia. 2. El paciente presenta un carcinoma broncognico no microctico estadio clnico IECB (T4N0M0). La realizacin de una ecoendoscopia esofgica que determine el grado de afectacin de la pared del esfago sera til para plantear una posible extirpacin, aunque se debe empezar con quimioterapia y radioterapia.

3. El paciente presenta un carcinoma broncognico no microctico estadio clnico IHB (T4N0M0). Se debe operar, extirpando todo el pulmn y la totalidad del esfago. 4. El paciente presenta un carcinoma broncognico no microctico estadio clnico fV (T4N0M0). La realizacin de una ecoendoscopia esofgica que determine el grado de afectacin de la pared del esfago sera til para plantear una posible extirpacin, aunque se debe empezar con quimioterapia y radioterapia. 5. El paciente presenta un carcinoma broncognico no microctico estadio clnico IHB (T4N0M0), por lo que en ningn caso es resecable. Se debe administrar tratamiento paliativo con quimioterapia. 51. Un paciente ingresado despus de un politraumatismo grave indica a las 24 horas disnea progresiva. Una primera radiografa de trax muestra infiltrados alveolares bilaterales que, pasadas unas horas, evolucionan a una imagen de "pulmn blanco". Cul es su diagnstico?: 1. 2. 3. 4. 5. Tromboembolismo pulmonar secundario a la inmovilizacin. Contusin pulmonar. Sndrome de distrs respiratorio del adulto. Hemorragia alveolar. Derrame pleural bilateral masivo por hemotrax. En relacin con la patologa del nervio ptico y los nervios oculomotores, es

52. cierto:

1. Un infarto de la corteza occipital produce una hemianopsia homnima ipsilateral. 2. La duracin ms habitual de la amaurosis fugaz es de entre 45 y 60 minutos. 3. En la neuritis ptica las pupilas son isocricas y puede encontrarse un defecto pupilar aferente. 4. La prdida de agudeza visual en el edema de papila es muy grave. 5. En las lesiones compresivas del III par (motor ocular comn) hay caractersticamente un respeto de la funcin pupilar 53. En qu situacin estara contraindicada la fibrinolisis endovenosa para tratar un infarto cerebral?: 1. 2. 3. 4. 5. Edad superior a setenta aos. Evolucin de la clnica de ms de dos horas. Historia de tratamiento hipotensor. Mejora espontnea del dficit neurolgico. TC cerebral normal.

54. En un paciente que presenta un cuadro agudo de cuadro confusional], oftalmoparesia por afectacin del sexto par bilateral y ataxia de la marcha pensara en: 1. 2. 3. 4. 5. Encefalopata hiperglucmica. Encefalopata de Korsakoff. Infarto cerebeloso. Administrarle inmediatamente tiamina. Intoxicacin por plomo.

55. En un periodo de 3 meses, un hombre de 42 aos ha presentado cuatro episodios de desconexin de su entorno, asociados con postura distnica de la mano derecha y movimientos de masticacin, de un minuto de duracin, que dando a continuacin confuso y con dificultad para la expresin verbal durante 10 minutos. Despus se recupera con normalidad pero no recuerda lo que le ha ocurrido. Cul es la actitud ms correcta?: 1. Iniciar tratamiento con clobazam oral y realizar un electroencefalograma (EEG). 2. Iniciar tratamiento con carbamacepina y realizar una resonancia magntica (RM) y un EEG. 3. Realizar un EEG y una RM de forma ambulatoria y esperar los resultados antes de iniciar el tratamiento. 4. Ingresar en la Unidad de Cuidados Intensivos e iniciar tratamiento con fenitona intravenosa. 5. Recomendar observacin por su familia y volver a revisin en tres meses. 56. Una joven de 13 aos sin antecedentes persona les de inters, presenta bruscamente mientras pasea con sus padres por un centro comercial, cuadro

vertiginoso asociado a vmitos. Inmediatamente despus, refiere parestesias en hemicuerpo derecho y disartria, que remiten en pocos minutos, y a continuacin cefalea, que mantiene a su llegada al centro de salud. Seale la respuesta correcta: 1. 2. 3. 4. 5. El diagnstico ms probable es el de sncope vasovagal. Si la exploracin fsica es normal no se requieren ms estudios. La ausencia de antecedentes personales excluye el diagnstico de migraa. El empleo de analgsicos y antiemticos no son aconsejables en este supuesto. Se debe evaluar la existencia de estmulos desencadenantes.

57. En un enfermo con una clnica de prdida de fuerza progresiva de 48 horas de evolucin que se inici a nivel distal, y ha ido ascendiendo progresivamente, una de las siguientes afirmaciones es FALSA: 1. Sospechara un sndrome de Guillain-Barr, 2. Vigilara la aparicin de complicaciones por neuropata autnoma. 3. Esperara encontrar disociacin albmina-citolgica a partir de los 10 das desde el inicio del cuadro. 4. Indicara tratamiento con Inmunoglobulinas intravenosas. 5. Los hallazgos neurofisiolgicos aparecen antes de que sea evidente la clnica. 58. Una mujer de 34 aos refiere diplopia de varias semanas. Unos meses antes haba sufrido diplopia, que remiti espontneamente. La exploracin muestra leve ptosis palpebral: 1. El sexo de la paciente hace improbable un diagnstico de Miastenia Gravis. 2. La presencia de lesiones desmielinizantes en la RMN cerebral confirmara que se trata de Miastenia Gravis. 3. En pacientes de Miastenia Gravis con afectacin ocular limitada, se observa presencia de Ac anti-receptores de acetilcolina en slo el 50%. Este dato positivo confirmara el diagnstico. 4. Se trata de un sndrome de Horner y la Rx de trax mostrara un tumor de Pancoast. 5. Los inmunosupresores no han demostrado eficacia en la Miastenia Gravis. 59. En una paciente de 68 aos con dolor facial en el que se sospecha una neuralgia idioptica de la segunda rama del nervio trigmino derecho, una de las siguientes respuestas NO sera correcta: 1. 2. 3. 4. 5. Hipoestesia en la zona malar derecha. La duracin del dolor es de unos pocos segundos. Los paroxismos dolorosos pueden desencadenarse al tocar suavemente una zona facial. El reflejo corneal es normal. Indicara tratamiento con carbamacepina.

60. Acude a urgencias un paciente de 80 aos que presenta desde hace 2 das alteracin del nivel de consciencia de forma fluctuante con lenguaje incoherente, agitacin y desorientacin. La familia refiere que durante la ltima semana al paciente se le ha prescrito lormetazepam por dificultad para conciliar el sueo. En la exploracin fsica el paciente se encuentra afebril con TA 140/70 y sin focalidad neurolgica motora ni sensitiva, con lenguaje incoherente y agitado. La frecuencia cardiaca era rtmica a 65 Ipm. Cul de las afirmaciones le parece correcta?: 1. Se trata de un paciente que ha sufrido un accidente vascular cerebral y debe realizarse una TAC cerebral urgente y administrar benzodiacepinas para controlar la agitacin. 2. Se trata de una clnica sugestiva de crisis comicial y debe realizarse un electroencefalograma urgente y administrar benzodiacepinas endovenosas. 3. Se trata de un delirium y debe retirarse el lormetazepam y administrar un neurolptico para controlar la agitacin. 4. Debera realizarse una puncin lumbar para descartar posible etiologa infecciosa. 5. Se trata de un efecto indeseable del lormetazepam que desaparecer al cabo de unos das sin ser necesario retirar medicacin. 61. La esclerosis mltiple es un proceso inflamatorio y desmielinizante del SNC. En el diagnstico de esta enfermedad es muy til la presencia de:

1. 2. 3. 4. 5.

Elevaciones del cido rico en plasma. Bandas oligoclonales en el lquido cefalorraqudeo. Ms de 100 linfocitos por microlitro en el lquido cefalorraqudeo. Cifras elevadas de cidos grasos de cadena muy larga en plasma. Ms de 50 polimorfonucleares por microlitro en el lquido cefalorraqudeo.

62. Cul de los siguientes frmacos cree que tiene ms posibilidades de inducir un parkinsonismo yatrgeno?: 1. 2. 3. 4. 5. Omeprazol. Cisapride. Risperidona. Clozapina. Quetiapina.

63. Ante un cuadro clnico de amenorrea- galactorrea y prdida de campo visual el primer diagnstico a considerar es: 1. Adenoma hipofisario no funcionante. 2. Prolactinoma. 3. Meningioma del tubrculo solar. 4. Pinealoma. 5. Intoxicacin por benzodiacepinas 64. Cul de las siguientes lesiones vasculares cerebrales conlleva un menor riesgo de sangrado intracraneal?: 1. 2. 3. 4. 5. Angioma venoso. Malformacin arteriovenosa. Cavernoma. Aneurisma mictico de la arteria cerebral media. Aneurisma sacular de la arteria comunicante anterior.

65. Qu actitud tomara ante un hombre de 25 aos con un ndulo tiroideo indoloro, de 4 cm. de tamao, de reciente aparicin, fro en la gammagrafa, con estudio hormonal normal, y con abundantes clulas foliculares en la puncin aspiradora con aguja fina?: 1. 2. 3. 4. 5. Levotiroxina a dosis sustitutivas y reevaluar a los 6 meses. Levotiroxina a dosis supresoras y reevaluar a los 3 meses. Tiroidectoma total. Administrar Yodo 131. Simplemente observar y repetir ecografa tiroidea a los 6 meses para valorar tamao.

66. Mujer de 88 aos, natural del Pirineo, donde siempre ha vivido, que consulta por un gran bocio multinodular junto con arritmia y hallazgo de un hipertiroidismo por T3 (T4 libre normal, T3 libre 8 praol/L, normal de 4 a 6.8, y TSH de 0.15 mUL, normal de 03 a 5). Reconoce que su bocio existe desde hace ms de 30 aos sin claro crecimiento reciente. No refiere disfagia ni estridor. Lleva tratamiento para hipertensin arterial, controlada con hidroclorotiacida y potasio, as como broncodilatadores y corticoides inhalados por una EPOC. Cul ser, probablemente, la actitud teraputica ms indicada?: 1. 2. 3. 4. 5. No tratar, ya que la T4 libre es normal. Hemitiroidectoma ms istmectoma. Iniciar L-tiroxina sdica. Iniciar propranolol. Tratamiento con yodo radioactivo.

67. En una paciente de 40 aos se encuentra una cifra de calcio de 11,8 mg/dl (normal menos de 10,5 mg/dl) en un anlisis realizado por otro motivo. La determinacin de PTH solicitada en el estudio de la hipercalcemia arroj una cifra de 115 pg/ml (lmite superior de la normalidad, 45 pg/ml). La densidad sea, el aclaramiento de creatinina y la calciuria son normales. Cul debe ser la conducta a seguir?:

1. 2. 3. 4. 5.

Administracin de bisfosfonatos. Paratiroidectoma. Administracin de quelantes del calcio. Observacin. Administracin de diurticos perdedores de calcio (furosemida).

68. Ante un paciente, con antecedentes de haber recibido hace 5 aos radioterapia sobre hipfisis, que llega a Urgencias estuporoso, con presin arterial sistlica de 70 mm Hg, fiebre de 39C y con sudoracin fra, lo primero que debe hacerse es: 1. 2. 3. 4. 5. Practicar cultivos (hemocultivos, urocultivos) para localizar el foco infeccioso. Realizar un test corto de Nuvacthen (ACTH) intravenoso. Colocar una va con cloruro sdico al 0,9% e hidrocortisona. Colocar una va con cloruro sdico al 0,9%, glucosa al 10% e hidrocortisona. Administrar hidrocortisona y tiroxina.

69. Cul es la enfermedad endocrina ms frecuente en la Neoplasia Endocrina mltiple tipo 1?: 1. 2. 3. 4. 5. Carcinoma medular de tiroides. Tumor endocrino del pncreas. Hiperparatiroidismo Primario. Feocromocitoma. Adenoma hipofisario. En relacin con los hipoglicemiantes orales es cierto que:

70. 1. 2. 3. 4. 5.

La acarbosa aumenta la secrecin de insulina. Las tiazolidindionas bloquean la a-glucosidasa intestinal. La metformina reduce la produccin heptica de glucosa. No pueden asociarse a la administracin de insulina. Con glibenclamida el riesgo de hipoglicemias es mnimo. La afectacin ms frecuente en la polineuritis diabtica es:

71. 1. 2. 3. 4. 5.

Proximal, bilateral, simtrica, dolorosa y motora Distal, bilateral, simtrica y motora. Distal, unilateral y motora. Distal, bilateral, simtrica y sensitiva. Proximal, bilateral, simtrica y sensitiva.

72. En relacin con las complicaciones que pueden acompaar a la acromegalia. Cul es la verdadera?: 1. Complicaciones neoproliferativas especialmente los osteosarcomas de huesos largos. 2. Complicaciones cardiovasculares. 3. Complicaciones neoproliferativas especialmente la neoplasia de pulmn. 4. Los aneurismas intracraneales se observan hasta un 70% de los casos. 5. A pesar de las complicaciones asociadas, la mortalidad no est aumentada en los casos de acromegalia no tratada. 73.Si una mujer de 35 aos consulta por presentar un microprolactinoma de 9 mm, podremos decirle que: 1. 2. 3. 4. 5. las Si permanece asintomtica y no presenta deseo gestacional no requiere tratamiento. S permanece asintomtica no requiere tratamiento ni seguimiento/observacin. An estando asintomtica debe iniciar tratamiento con agonistas dopaminrgicos. El tratamiento quirrgico es recomendable de entrada en lesiones prximas a los 10 mm. Si presenta sntomas, la radioterapia es una alternativa de tratamiento dado que se normalizan cifras de prolactina en un 80% de los casos. Cul de las siguientes contraindica la nutricin parenteral?:

74.

1. 2. 3. 4. 5.

Pancreatitis aguda grave. Fstula yeyunal. Reseccin subtotal de intestino delgado Expectativa de ayuno de menos de 5 das. Postoperatorio de hemicolectoma.

75. Cul de los siguientes NO es un efecto secundario frecuente del tratamiento con estatinas?: 1. Aumento de la TSH. 2. Aumento de la CPK (Creatinfosfoquinasa). 3. Aumento de las transaminasas. 4. Mialgias. 5. Artralgias 76. En relacin al aumento de la prevalencia de la obesidad y la inactividad fsica en la poblacin, se ha incrementado la frecuencia de un trastorno denominado sndrome metablico. Seale cual de las siguientes caractersticas NO forma parte del sndrome metablico: 1. 2. 3. 4. 5. Hipertrigliceridemia. Hipertensin arterial. Hiperglucemia en ayunas Aumento del permetro de la cintura abdominal Aumento de la concentracin plasmtica de colesterol-LDL.

77. Cul de las siguientes afirmaciones que reflejan las caractersticas clnicas de la artritis reumatoide es correcta?: 1. 2. 3. 4. 5. La afectacin de la columna lumbar es ms frecuente que la cervical. La afectacin de las articulaciones interfalngicas distales es ms frecuente que las proximales. La artritis reumatoide es ms frecuente en los hombres en reas rurales. Es una poliartritis de ms de 6 semanas de duracin. La artritis reumatoide del adulto se asocia con un aumento de la frecuencia de uvetis.

78. Paciente de 40 aos con febrcula y sndrome constitucional (astenia, anorexia, prdida de peso), que presenta epistaxis recurrente con dolor en tabique nasal. Se realiza una radiografa de trax en la que se aprecian infiltrados pulmonares nodulares cavitados bilaterales. Se practica una analtica con estos resultados: 12.000 leucos/mm3 con 68% neutrfilos (7.800 por mm3), 23% linfocitos (2760 por mm3), 5% monocitos (600 por mm3) y 1% eosinfilos (120 por mm3), creatinina normal, anticuerpos anti citoplasma de neutrfilo positivos (c-ANCA). Qu entidad sospechara en este paciente?: 1. 2. 3. 4. 5. Poliarteritis Nodosa clsica. Granulomatosis alrgica de Churg-Strauss. Granulomatosis de Wegener. Arteritis de Takayasu. Artritis Reumatoide.

79. Cul de las siguientes manifestaciones es compartida por las espondiloartropatas?: 1. 2. 3. 4. 5. La inflamacin ocular (uvetis, conjuntivitis). La leucopenia. La hipertensin arterial, La presencia de afectacin gstrica. La afectacin renal.

80. En una paciente de 68 aos con arteritis de clulas gigantes. Cul de los siguientes datos de laboratorio suele estar menos alterado?: 1. Hemoglobina.

2. 3. 4. 5.

Recuento de leucocitos. Hierro srico. Alfa 2 globulinas. Velocidad de sedimentacin globular.

81. Ante un paciente con sospecha de Poliarteritis Nodosa (PAN). Cul de las siguientes pruebas complementarias diagnsticas tiene mayor rendimiento?: 1. 2. 3. 4. 5. Complemento srico. Arteriografa abdominal Determinacin de anticuerpos anticitoplasma del neutrfilo (ANCA). Endoscopia digestiva alta. Medida del flujo lagrimal.

82. Cul de estas afirmaciones es FALSA con res pecto a las artritis inducidas por microcristales?: 1. Pueden ser producidas por cualquiera de los siguientes cristales: urato monosdico, pirofosfato clcico, hidroxiapatita clcica y oxalato clcico. 2. El depsito de cristales puede producir cuadros clnicos similares a la artritis reumatoide o la espondilitis anquilosante. 3. Los cuadros clnicos producidos son especficos para cada uno de los tipos de cristales depositados. 4. Para hacer el diagnstico es imprescindible el estudio del lquido sinovial con microscopio de luz polarizada para identificar el tipo de cristales. 5. El lquido sinovial suele ser de tipo inflamatorio, aunque en ocasiones existen cristales en ausencia de inflamacin. 83. Una mujer de 58 aos con antecedentes de diabetes mellitus tipo 1 y artrosis de ambas rodillas acude al servicio de urgencia por fiebre de hasta 39C junto con dolor y tumefaccin de 24 horas de evolucin en la rodilla derecha. Cul de las siguientes actitudes o exploraciones complementarias est ms indicada?: 1. Artrocentesis, examen del lquido sinovial con microscopio de luz polarizada y test de Gram urgente. 2. Tratamiento con antibiticos intraarticulares. 3. Estudio inmunolgico, incluyendo factor reumatoide. 4. Lavado articular y tratamiento con antibiticos intravenosos. 5. Artrocentesis e infiltracin con corticosteroides de accin prolongada. 84. Una mujer de 84 aos acude a nuestra consulta por dolor crnico, a veces intenso, en la zona media de la columna vertebral torcica, que aumenta al sentarse y levantarse. A la exploracin se detecta una marcada cifosis y contractura de la musculatura paravertebral. Tambin se palpa una masa pulstil en el epigastrio. Cul es la causa ms probable del dolor?: 1. 2. 3. 4. 5. Aneurisma abdominal con erosin vertebral acompaante. Estenosis de canal lumbar. Hernia de disco aguda. Osteoporosis con aplastamiento vertebral. Enfermedad de Paget

85. Paciente de 40 aos que acude al Servicio de Urgencias tras sufrir un traumatismo directo sobre el brazo izquierdo al caerse de la motocicleta. La exploracin clnica es la siguiente: dolor, movilidad anormal e impotencia funcional en la zona media del brazo, pulsos distales humeral y radial presentes, imposibilidad para realizar la extensin activa de la mueca y de dos, e hipoestesia en la zona dorsal del primer dedo y del primer espacio interdigital. Las radiografas muestran una fractura conminuta del tercio medio del hmero. Cul es la lesin asociada ms probable que presenta?: 1. 2. 3. 4. Lesin arteria humeral y nervio mediano. Lesin nervio cubital. Lesin nervio radial. Lesin nervio mediano.

5. Lesin nervios radial y mediano. 86. Un chico de 20 aos, jugando al ftbol hace una semana, sufri una torsin en su rodilla al clavar los tacos de la bota en el csped y girar bruscamente el tronco. A la exploracin no se aprecia derrame articular, la palpacin del tercio posterior de la interlnea articular medial es dolorosa, duele al forzar la flexin, la maniobra de Lachman es negativa y los bostezos a varo y valgo son indoloros. Cul es el diagnstico ms probable?: 1. Lesin meniscal del menisco medial. 2. Lesin capsular posteromedial. 3. Lesin del ligamento cruzado anterior. 4. Lesin del complejo ligamentario posterolateral. 5. Lesin del ligamento colateral me 87. Trabajando como mdico rural recibimos, en la urgencia de un centro de salud situado a 100 Km del hospital, una fractura abierta de tobillo por cada en la montaa, presentando una contaminacin grave por restos vegetales y tierra. Nuestra acritud teraputica debe ser: 1. Nula, evacuacin inmediata solicitando transporte areo. 2. Antibioterapia, profilaxis antitetnica, retirada de contaminacin grosera, lavado inicial, cobertura y estabilizacin provisional para traslado inmediato. 3. Antibioterapia, profilaxis antitetnica, retirada de contaminacin grosera, sutura de aproximacin para cierre de la herida, inmovilizacin provisional y traslado inmediato. 4. Antibioterapia, profilaxis antitetnica y traslado inmediato. 5. Retirada de contaminacin grosera, lavado inicial y estabilizacin provisional para traslado inmediato. 88. Acude a la urgencia una mujer de 53 aos tratada dos das antes con una reduccin cerrada y yeso antebraquial cerrado por una fractura de Colles en su mueca izquierda. Aqueja dolor muy intenso y progresivo en mano y mueca, as .como incapacidad de mover los dedos y falta de sensibilidad en ellos. El yeso presenta un aspecto adecuado. Los dedos presentan buen relleno capilar y una coloracin aceptable, pero estn muy hinchados, con nula movilidad activa, y su movilizacin pasiva produce intenso dolor. La actitud ms correcta ser: 1. Mantener el yeso, administrar antiinflamatorios y remitir a la paciente a consultas externas. 2. Mantener el yeso, elevar la mano, ingresar a la paciente en observacin instndole a mover activamente los dedos. 3. Abrir el yeso, antiinflamatorios y diurticos, estimular la movilidad activa, y si no mejora rpidamente osteosntesis de la fractura. 4. Abrir el yeso, administrar antiinflamatorios y remitir a la paciente a consultas externa. 5. Abrir el yeso, antiinflamatorios y diurticos, estimular la movilidad activa, y si no mejora rpidamente fasciotoma urgente. 89. Una auxiliar de clnica de 40 aos de edad acu de a nuestra consulta por dolor intenso en el codo derecho, irradiado por cara anterolateral de antebrazo, de 4 das de evolucin, que le dificulta sus actividades cotidianas y laborales. La movilidad activa del codo es completa, sien do dolorosa la palpacin del origen de los msculos radiales, El dolor empeora con flexin de codo contra resistencia y con extensin de mu eca contra resistencia. La radiografa simple es rigurosamente normal. Nuestra actitud inicial deber ser: 1. 2. 3. 4. 5. Serie de 3 infiltraciones con corticoide y anestsico local. Antiinflamatorios, codera con dispositivo de presin, aplicacin de calor local. Resonancia magntica. Inmovilizacin con yeso braquial. Indicacin quirrgica inmediata.

90. Hombre de 29 aos: accidente de trfico. Ingresa en servicio de Traumatologa con cuadro de prdida progresiva de fuerza en miembros inferiores, los reflejos osteotendinosos estn abolidos. El estudio con radiografas simples muestran una fractura por compresin de L1 con desplazamiento del muro posterior y acuamiento

anterior de un 50%, Qu prueba diagnstica indicara para valorar la ocupacin del canal raqudeo?: 1. 2. 3. 4. 5. Una termografa. Una densitometra sea. Una TAC vertebral centrada en regin dorso lumbar. Una gammagrafa sea. Una tomografa cervical.

91. Tras un accidente de coche llega al Servicio de Urgencias una paciente de 34 aos inconsciente, hipotensa, disneica y con fracturas abiertas en ambas extremidades inferiores. En la valoracin inicial o revisin primaria rpida y resucitacin, usted debe realizar el llamado protocolo ABCDE de la atencin al paciente politraumatizado. Este incluye los siguientes pasos, EXCEPTO uno, indique este ltimo: 1. 2. 3. 4. 5. Mantenimiento va area con control de la columna cervical. Anlisis del dficit neurolgico. Tratamiento de las fracturas abiertas. Desvestir completamente a la paciente y prevenir la hipotermia. Anlisis del estado circulatorio.

92. Un paciente politoxicmano de 28 aos ingres hace 3 das por fractura subtrocantrea de fmur izquierdo, fractura de rtula derecha y fractura diafisaria conminuta de tibia izquierda que se inmovilizaron provisionalmente en espera de ciruga de osteosntesis. Bruscamente inicia un cuadro de estupor y obnubilacin intensos acompaado de disnea y de aparicin de petequias difusas. Debemos sospechar: 1. 2. 3. 4. 5. Neumona nosocomial por encamamiento. Shock hipovolmico. Coma exotxico por abuso de sustancias ilegales. Shock neurognico por dolor. Sndrome de embolia grasa.

93.Un hombre de 66 aos, diagnosticado de hiperplasia benigna de prstata, presenta un cuadro de fiebre con escalofros. El examen de orina muestra que su pH es 8,5. El sedimento urinario contiene cristales de estruvita (MgNHjPOj). Seale la respuesta correcta: 1. 2. 3. 4. 5. Padece una acidosis tubular que le impide acidificar la orina. El pH urinario, normal en este paciente, descarta una infeccin. Cualquier proceso infeccioso bacteriano urinario, eleva el pH. Debe sospecharse una infeccin urinaria por grmenes que degradan la urea. El pH bsico y la presencia de cristales triples, sugieren infeccin por cndidas.

94. Un paciente de 52 aos con el diagnstico de ulcus pilrico y vmitos de repeticin de una semana de duracin acude al Hospital con una tensin arterial de 100/58 mmHg y la siguiente analtica: Plasma: Na+ 140 mmol/l, K+ 2.2 mmol/l, Cl" 86 mmol/l, C03H* 42 mmol/l, pH 7.53, pC02 53 mmHg y creatinina 2,9 mg/dl; Orina Na+ 2 mmol/l, C 21 mmol/l, pH 5. Cul de los siguientes diagnsticos es cierto?: 1. 2. 3. 4. 5. 95. 1. 2. 3. 4. 5. Alcalosis mixta. Acidosis metablica con vaco amnico normal Alcalosis metablica. Alcaluria paradjica, Acidosis hipoclormica. Qu es cierto acerca de la oliguria?: Se define como un volumen urinario inferior a 1.000ml/da. Es un hallazgo constante en la insuficiencia renal aguda. Se asocia a dolor lumbar bilateral. Hace ms difcil el manejo conservador del fracaso renal agudo. Mejora el pronstico del fracaso renal agudo.

96.

Seale la respuesta correcta referida a la Prpura de Henoch-Schnlein:

1. La lesin glomerular, idntica a la de la Neuropata IgA (enfermedad de Berger), se caracteriza por depsitos de IgA en el mesangio glomerular. 2. Es una entidad exquisitamente sensible al tratamiento con corticoides. 3. Se detectan Ac anticitoplasma de neutrfilos en 80% de pacientes. 4. Debe sospecharse en pacientes con hemoptisis e insuficiencia renal aguda. 5. La presencia de proteinuria masiva es clave en el diagnstico de esta enfermedad 97. Una mujer de 24 aos acude a urgencias remitida por su mdico de Atencin Primaria porque al realizarle una analtica por astenia, artralgias, febrcula y aparicin de edemas maleolares, objetiva anemia normoctica normocrmica (hemoglobina de 9 gs/dl), creatinina srica de 2 mgs/dl, sedimento con microhematuria y proteinuria en tira reactiva de 500 mgs/dl. Cul o cules seran las exploraciones complementarias que usted solicitara en primer lugar dado el diagnstico de presuncin?: 1. 2. 3. 4. 5. Cuantificacin de proteinuria en orina de 24 horas y aclaramiento de creatinina. Tomografa axial computarizada renal. Test de Coombs, haptoglobina, ferrocintica. C3, C4, ANA, AntiDNA. Biopsia renal.

98. Cul es la causa principal de muerte en pacientes con insuficiencia renal crnica, sometidos a tratamiento con hemodilisis?: 1. 2. 3. 4. 5. Hiperpotasemia. Imposibilidad de dilisis por falta de acceso vascular. Sepsis de punto de partida en acceso vascular. Hemorragias relacionadas con el uso de heparina. Enfermedad cardiovascular.

99. Seale cul de las siguientes medidas NO es adecuada para evitar la progresin de la nefropata diabtica: 1. Restriccin de protenas en la dieta. 2. Control glucmico estricto en los diabticos tipo 1. 3. Control glucmico estricto en los diabticos tipo 2, 4. Incremento de la presin de perfusin glomerular. 5. Tratamiento con inhibidores de la enzima conversora de angiotensina, en pacientes con microalbuminuria. 100. Cul de estas sustancias es habitualmente utilizada como marcador tumoral de los tumores de testculo?: 1. 2. 3. 4. 5. CEA. PSA. AFP. TPA. CA 19.9.

101. En un paciente con un traumatismo plvico y sospecha de rotura uretral por incapacidad de orinar espontneamente, globo vesical y uretrorragia. Cul de estas maniobras debe evitarse?: 1. 2. 3. 4. 5. Sondaje vesical. Colocacin de cistostoma suprapbica. Realizacin de uretrografa retrgrada. Tacto rectal. Palpacin abdominal.

102. Qu diagnstico debe sospecharse en un hombre de 53 aos, fumador, que inicia de manera insidiosa un sndrome miccional irritativo con tacto rectal normal, flujo urinario no obstructivo, ecografa reno-vsico-prosttica normal, sedimento urinario con

microhematuria y urocultivo negativo, cistoscopia normal y citologa urinaria con atipias?: 1. 2. 3. 4. 5. Prostatitis. Litiasis uretral. Carcinoma in situ vesical. Hipernefroma. Esquistosomiasis.

103. El carcinoma de clulas renales del adulto suele asociarse con mayor frecuencia a: 1. 2. 3. 4. 5. Adenocarcinoma de pncreas. Adenocarcinoma de endometrio. Insuficiencia renal crnica adquirida y dilisis. Linfoma. Quimioterapia previa con ciclofosfamida.

104. Una maestra jubilada de 74 aos con diabetes mellitus presenta frecuentes episodios de retencin urinaria. No presenta incontinencia al toser o cambiar de postura. En la exploracin fsica se detecta una neuropata sensitiva en extremidades inferiores. El residuo postmiccional es de 400 mL. La maniobra de Valsalva realizada en bipedestacin no provoca prdida de orina. El estudio urodinmico muestra un detrusor acontrctil sin obstruccin al vaciamiento vesical. Los estudios de laboratorio son normales y el tratamiento farmacolgico ha sido inefectivo. Cul es la opcin teraputica ms apropiada?: 1. 2. 3. 4. 5. Sondajes intermitentes. Sonda urinaria permanente. Dilatacin uretral. Uretrolisis. Ureterotoma transuretral.

105. Ante un paciente con insuficiencia renal crnica y sospecha de uropata obstructiva en una exploracin ecogrfica. Qu exploracin es la idnea para evaluar el nivel de obstruccin y la causa?: 1. 2. 3. 4. 5. Renograma isotpico. Cistoscopia, Tomografa axial computarizada, Urografa mediante resonancia magntica. Urografa intravenosa.

106. Cul cree usted que es el trastorno endocrino que provoca mayor prevalencia de disfuncin erctil?: 1. 2. 3. 4. 5. Diabetes Mellitus. Sndrome de deficiencia andrognica asociada a la edad. Hipogonadismo hipogonadotropo. Sndrome de Cushing. Hipotiroidismo.

107. Seale cul de las siguientes afirmaciones NO es correcta en relacin con la aplasia medular: 1. Cursa con pancitopenia. 2. Se trata con trasplante de mdula sea. 3. Puede responder al suero antitimoctico (ATG). 4. Cursa con eritroblastos en sangre perifrica. 5. Los reticulocitos estn descendidos 108. Hombre de 27 aos, con antecedentes de enfermedad de Crohn y reseccin de leon terminal hace 3 aos, presenta intensa astenia y palidez cutneo-mucosa. En la bioqumica destaca LDH 2730 UI/L y bilirrubina 1,7 mg/dL. En el hemograma muestra 2,9 x 109 leucocitos/L con re cuento diferencial normal, hemoglobina 7,5 g/dL, hematocrito

22%, VCM 135 fl y 105 x 109 plaquetas/L. El recuento de reticulocitos es 1% y en el frotis de sangre perifrica se observan abundantes neutrfilos hipersegmentados. Cul sera su juicio diagnstico ms probable y su actitud teraputica inicial?: 1. Anemia de probable origen inflamatorio. Transfundir concentrado de hemates. 2. Anemia megaloblstica por dficit de vitamina B12. Iniciar vitamina B12 parenteral y cido flico oral 3. Anemia hemoltica. Iniciar corticoides. 4. Sndrome mielodisplsico. Transfundir concentrado de hemates. 5. Anemia megaloblstica por dficit de cido flico. Iniciar cido flico oral. 109. Una chica de 27 aos, asintomtica, le consulta porque en una revisin de la empresa le han detectado alteraciones analticas. Aporta los siguientes resultados: hemates 4.9 x 106/L, hemoglobina 9 g/dl, VCM 75 fl, leucocitos 6.2 x 109/L, plaquetas 220 x 109/L, bilirrubina total 12 mmol/1 (normal: 5 - 17), LDH 2.8 (normal: 1.7 - 3.2), sideremia 20 jj.mol/1 (normal: 9 - 27) y ferritina 180 U/I (normal: 10 - 200). Cul es el diagnstico ms probable?: 1. 2. 3. 4. 5. Anemia ferropnica. Anemia hemoltica. Anemia sideroblstica. Mielofibrosis. Beta-talasemia.

110. Ante los siguientes hallazgos analticos: Hemoglobina 8,5 gr/dL, VCM 85 fl, Bilirrubina normal, hierro srico 10 pg/dL, capacidad de fijacin total de hierro 200 U/dL, ndice de saturacin de la transferrina 15% y ferritina 150 pg/dL. Qu tipo de anemia pensara que tiene el paciente?: 1. 2. 3. 4. 5. Anemia por dficit de vitamina B12. Anemia inflamatoria (de proceso crnico). Anemia ferropnica. Anemia por dficit de cido flico. Anemia hemoltica aguda.

111. Cul es el tratamiento de primera lnea de un paciente con cifras de plaquetas inferiores a 10 x 109/L, ditesis hemorrgica y un aspirado de mdula sea con abundantes megacariocitos?: 1. 2. 3. 4. 5. Ciclosporina. Prednisona. Hidrocortisona. Esplen Inmunoglobulinas intravenosas.

112. En una paciente de 20 aos de edad con adenopatas laterocervicales dolorosas tras la ingesta de alcohol, fiebre y prurito intenso generalizado el diagnstico ms probable sera: 1. 2. 3. 4. 5. Mononucleosis infecciosa. Toxoplasmosis. Enfermedad de Hodgkin. Linfoma difuso de clulas grandes. Tuberculosis ganglionar.

113. Mujer de 30 aos que consulta porque en una revisin de empresa le han detectado un hemograma normal, con leucocitos 35 x 109/L (60% segmentados, 12% cayados, 16% mielocitos, 4% meta mielocitos, 7% linfocitos, 1% monocitos, hemoglobina 127 g/L, VCM 89 fL, HCM 28 pg, reticulocitos 46 x 109/L y plaquetas 389 x 109/L. La morfologa eritrocitaria es normal. En el resto de anlisis destaca un cido rico de 8 mg/dL y una LDH de 650 UI/L. Su estado general est conservado, sin ningn otro sntoma que una ligera sensacin de astenia desde hace 2-3 meses. No ha perdido peso, no tiene fiebre ni ningn sntoma de infeccin ni de dolor. La exploracin fsica muestra la presencia de una esplenomegalia de 1-2 traveses de dedo por debajo del

reborde costal como nico hallazgo. La paciente no es fumadora y no tiene antecedentes de inters excepto que hace seis meses se le practic otro hemograma que mostr una cifra leucocitaria de 14 x 109/L, que se atribuy a una infeccin respiratoria que resolvi sin problemas. Cul es el diagnstico ms probable en ste caso?: 1. 2. 3. 4. 5. Leucocitosis reactiva. Leucemia mieloide crnica (LMC). Mielofibrosis en etapa incipiente. Leucemia mielomonoctica crnica (LMMC). Sndrome mielodisplsico.

114. Un paciente VIH positivo presenta una gran masa abdominal, adenopatas a nivel supra e infradiafragmtico, sudoracin nocturna y fiebre. La biopsia ganglionar muestra una proliferacin difusa de clulas de mediano tamao no hendidas, con un citoplasma intensamente basfilo con vacuolas, CD19 y CD20 positivas, y reordenamiento del protooncogn C-MYC. De entre las siguientes entidades cul es el diagnstico ms probable: 1. 2. 3. 4. 5. Linfoma folicular. Enfermedad de Hodgkin. Linfoma de Burkitt. Linfoma de clulas del manto. Linfoma T hepatoesplnico.

115. Hombre de 65 aos diagnosticado de mieloma mltiple IgG Kappa con un nivel de paraprotena de 2.400 mg/dl, sin proteinuria de Bence Jones, anemia, insuficiencia renal, hipercalcemia ni lesiones seas significativas. La actitud teraputica inicial debe ser: 1. 2. 3. 4. 5. Tratamiento con esquema Melfaln y Prednisona. Esquema Melfaln, Prednisona y Talidomida. Autotrasplante de progenitores hematopoyticos. Poliquimioterapia tipo VAD. No tratamiento inicial, seguimiento e iniciar tratamiento ante datos de progresin.

116. Cul de las siguientes afirmaciones sobre el tratamiento de la trombosis venosa profunda (TVP) es FALSA?: 1. En pacientes con TVP confirmada objetivamente, recomendamos tratamiento agudo con heparina de bajo peso subcutnea o heparina no fraccionada. 2. Para pacientes con elevada sospecha clnica de trombosis, se recomienda tratamiento anticoagulante mientras se espera el resultado de las pruebas diagnsticas. 3. El tratamiento con heparina se dice debe controlarse realizando TPT (tiempo parcial de tromboplastina). 4. Se recomienda iniciar la administracin de anticoagulantes orales junto con heparina en el primer da del tratamiento, e interrumpir la administracin de heparina cuando el INR sea estable y superior a 2. 5. En los pacientes tratados con heparina de bajo peso es necesario hacer mediciones sistemticas de la actividad antiXa con el fin de ajustar la dosis 117. Hombre de 76 aos con antecedentes de hipertensin arterial en tratamiento con enalapril que consulta por presentar de forma brusca disartria y hemiparesia derecha que desaparece en doce horas. Eco-Doppler de troncos supraarticos: ateromatosis carotdea bilateral con estenosis del 55% en la cartida izquierda, y del 30% en la derecha. Electrocardiograma: fibrilacin auricular con respuesta ventricular a 90 Ipm. TAC craneal: normal. Para la prevencin de nuevos episodios su tratamiento inicial sera: 1. 2. 3. 4. 5. Acenocumarol, ajustndolo a la dosis que consiga un INR de 2.5. cido acetil saliclico: 300 mg al da. Clopidogrel: 75 mg al da. Angioplastia transluminal percutnea carotdea izquierda. Endarterectoma carotdea izquierda.

118. Paciente de 36 aos que, debido a una hematemesis masiva por un ulcus recibe una transfusin con 2U de concentrado de hemates. A los 5-10 m. de iniciarse la transfusin comienza con fiebre, escalofros, hipotensin, dolor en regin lumbar y oliguria. Cul sera el diagnstico ms probable?: 1. 2. 3. 4. 5. Contaminacin bacteriana de la sangre. Sepsis secundaria al ulcus. Reaccin febril secundaria a la transfusin. Reaccin febril por el plasma que contamina los hemates. Reaccin transfusional hemoltica.

119. Acude a nuestra consulta un paciente de 30 aos con fiebre, cefalea y "trancazo" desde hace 48 horas. En la exploracin, adems de fiebre de 38,5C, se observa un exantema maculo-papuloso generalizado que afecta a palmas y plan tas y una pequea lesin costrosa en una pierna. Cul de los siguientes es el diagnstico ms probable?: 1. 2. 3. 4. 5. Fiebre botonosa. Les secundaria. Tifus murino. Mononucleosis infecciosa. Sndrome de Stevens-Johnson.

120. Cul de las siguientes relaciones entre microorganismo y enfermedad es INCORRECTA?: 1. 2. 3. 4. 5. Arbovirus - dengue. Rickettsias - tifus. Protozoos - esquistosomiasis. Hongos - histoplasmosis. Bacterias - lepra.

121. Los grmenes que con mayor frecuencia infectan las heridas quirrgicas limpias son: 1. 2. 3. 4. 5. Gnero Staphylococcus. Enterobactericeas. Bacteroides sp. Pseudomona aeruginosa. Streptococcus sp.

122. Un hombre de 43 aos, ingeniero de profesin, ha permanecido durante 2 meses en la RD del Congo. A los 8 das de su regreso, comienza con un cuadro de fiebre, escalofros, cefalea, mialgias, nauseas y dolor abdominal. A la exploracin: esplenomegalia y discreta hepatomegalia. En la analtica sangunea destaca: Hb 9,8 gr/dl; Leucocitos 3465 mm3, plaquetas 97.000, AST 121, ALT 119 y Na 129. Es FALSO sobre esta patologa que: 1. 2. 3. 4. 5. Para su diagnstico es til un frotis y una gota gruesa. Podra haberse evitado con una vacunacin correcta. Es una enfermedad potencialmente mortal. Las medidas de barrera son fundamentales en la prevencin. La doxiciclina ha demostrado eficacia en su profilaxis.

123. Hombre de 47 aos, natural de Brasil, residente en Espaa desde hace 10 aos, acude a su consulta por presentar disfagia y estreimiento crnico. Entre sus antecedentes destaca, ser portador de marcapasos y una miocardiopata dilatada. En el trnsito gastroesofgico se objetiva un megaesfago, as como megacolon en el enema opaco. La prueba diagnstica ms rentable para el diagnstico sera: 1. 2. 3. 4. Panendoscopia oral y colonoscopa para toma de biopsias. Aspirado de mdula sea para Leishmania sp. Serologa para T. cruzi. Coprocultivo para enteropatgenos.

5. PCR para malaria. 124. Sealar la respuesta correcta en relacin con la osteomielitis vertebral:

1. La forma ms comn de afectacin vertebral es la diseminacin por contigidad de un foco paramenngeo. 2. El agente causal ms frecuente es Escherichia Coli. 3. La resonancia magntica es el procedimiento diagnstico de eleccin. 4. La evolucin slo con tratamiento mdico es generalmente desfavorable. 5. La duracin recomendable del tratamiento antibitico es de 4 semanas. 125. Las pautas cortas (3 das) de tratamiento anti bitico de la infeccin urinaria, NO se aconsejan en el caso de: 1. 2. 3. 4. 5. Infeccin por Proteus Mirabilis. Gnero masculino Presencia de ms de 105 unidades formadoras de colonias por microlitro de cualquier bacteria. Infecciones por Stafilococcus saprophyticus. Infeccin por Escherichia Coli.

126. Un paciente de 80 aos con antecedentes de insuficiencia renal crnica y diabetes tipo 2, presenta desde hace 3 das tos con expectoracin purulenta, fiebre alta, dificultad respiratoria y dolor costal derecho. En la exploracin presenta una presin arterial de 120/60 mmHg, frecuencia cardiaca de 80 LPM, y respiratoria de 20 RPM. La temperatura es de 38C; se encuentra algo confuso y parcialmente desorientado. Presenta crepitantes localizados en la base pulmonar derecha y la Rx de trax muestra un infiltrado de pequeo tamao en esa localizacin. Cul de las siguientes es la opcin de tratamiento antibitico ms adecuada?: 1. 2. 3. 4. 5. Eritromicina IV ms Cefuroxima IV, hospitalizado, Amoxicilina-clavulnico IV ms Gentamicina IV, hospitalizado. Ceftriaxona IV ms Claritromicina VO, hospitalizado. Telitromicina IV hospitalizado. Levofloxacino durante 7 das, ambulante.

127. Entre los principios bsicos del tratamiento de la tuberculosis NO se encuentra uno de los siguientes: 1. La utilizacin de ms de un frmaco al que el microorganismo sea sensible. 2. La estrategia de induccin con un nmero elevado de frmacos durante dos meses, seguida de un tratamiento simplificado de duracin prolongada. 3. En la actualidad el tratamiento de inicio recomendado est compuesto por isoniacida, rifampicina, etambutol y pirazinamida. 4. En casos de fracaso teraputico se recomienda monitorizar los niveles plasmticos de los frmacos. 5. La utilizacin de procedimientos de administracin directamente observada es recomendable en determinados paciente 128. En relacin al tratamiento de la Sfilis. Cul de las siguientes es la respuesta correcta?: 1. 2. 3. 4. 5. No hay ningn tratamiento efectivo. La Penicilina es el tratamiento de eleccin en cualquier estadio. La Penicilina es el tratamiento de eleccin slo en la Neurosfilis. La Penicilina es el tratamiento de eleccin slo en la Sfilis no complicada. El tratamiento de eleccin son las Tetraciclinas.

129. Un funcionario de prisiones previamente sano, no fumador, de 35 aos, con tos y expectoracin mucoide desde hace 3 meses, acude a consulta con una radiografa reciente que muestra un infiltrado cavitado en lbulo superior derecho. Est eupneico, no tiene fiebre ni dolor alguno. El examen fsico es normal. Lo ms adecuado ser ahora:

1. Recoger al menos 3 muestras de esputo matutino para baciloscopia y cultivo, solicitar Mantoux y anlisis de sangre, incluyendo transaminasas, y serologa de VIH, y aislarle en domicilio hasta tener los resultados. 2. Avisar a Epidemiologa de la Comunidad Autnoma para que inicie el protocolo de estudio y tratamiento de los contactos. 3. Remitirle a Urgencias del Hospital para estudio y tratamiento antituberculoso, durante 2-3 semanas, o hasta que la baciloscopia sea negativa. 4. Comenzar tratamiento antituberculoso mientras esperamos las baciloscopias de esputo. 5. Dado que es una persona de alto riesgo, debe estar vacunado de la tuberculosis y por ello el Mantoux no es valorable. 130. Seale la correcta en relacin con la circuncisin y la prevencin de la transmisin de la infeccin por VIH: 1. En frica se ha mostrado eficaz en la prevencin de la transmisin de la mujer al hombre. 2. Reduce la transmisin tras relaciones homosexuales entre hombres. 3. Incrementa la transmisin del VIH del hombre a la mujer. 4. El balance coste-beneficio es desfavorable en pases subdesarrollados. 5. Se recomienda generalizar el procedimiento para reducir las tasas de transmisin en todo el mundo. 131. Cul sera el tratamiento quirrgico de eleccin para una lesin cutnea en regin deltoidea con Anatoma Patolgica de Melanoma Maligno Clarck IV y Breslow de 1,92 cm: 1. 2. 3. 4. 5. Ampliacin de mrgenes de 1 cm. Ampliacin de mrgenes de 2 cm si la Ecografa axilar es negativa. Ampliacin de mrgenes de 2 cm y deteccin del ganglio centinela. Ampliacin de mrgenes de 2 cm y vaciamiento axilar. Interfern adyuvante previo al tratamiento quirrgico.

132. Mujer de 60 aos de edad diagnosticada de carcinoma ductal infiltrante multicntrico de mama subsidiaria de mastectoma ms vaciamiento axilar y quimioterapia postoperatoria. Cundo estara indicado realizar la reconstruccin de la mama?: 1. Sistemticamente diferida al cabo de 5 aos libre de enfermedad postmastectoma. 2. El momento {inmediato-diferido) vendr indicado por el nmero de ganglios positivos. 3. La quimioterapia postoperatoria es una contraindicacin para realizar la reconstruccin inmediata. 4. De forma inmediata, pues la reconstruccin de la mama no interfiere en la historia natural de la enfermedad. 5. A partir de los 60 aos es mejor el uso de prtesis externa. 133. Una mujer de 33 aos consulta por un ndulo mamario que se diagnostica de carcinoma ductal. Su madre a los 50 aos y su hermana a los 40 han tenido cncer de mama. Su ta materna a los 45 y su abuela a los 55 han tenido cncer de ovario. Qu estudio solicitara?: 1. 2. 3. 4. 5. BCRA1. Her2. P53. Receptor de estrgeno. Receptor de progesterona.

134. Una mujer de 70 aos consulta por astenia y anemia ferropnica que no presentaba en anlisis del ao anterior. En el estudio se demuestra un cncer intestinal. En qu lugar estar ms frecuentemente localizado?: 1. 2. 3. 4. 5. Duodeno. Yeyuno. Ciego. Sigma. Recto.

135. Un paciente con carcinoma de pulmn de clulas escamosas inoperable desarrolla una hipercalcemia de 14 mg/dl. Se inicia una perfusin con suero salino y, una vez bien hidratado, furosemida, con lo que horas ms tarde la calcemia es de 11,5 mg/dl. Cul es el paso teraputico siguiente ms apropiado?: 1. 2. 3. 4. 5. Corticoides. Nitrato de galio. Alendronato. Plicamicina. Zoledronato intravenoso.

136. Con respecto a la utilizacin de analgsicos opioides en enfermos con cncer en situacin paliativa. Indique la respuesta INCORRECTA: 1. El estreimiento es un efecto secundario casi constante por lo que se debera utilizar un laxante para evitar su aparicin. 2. Su uso continuado requiere aumentar la dosis administrada para obtener el mismo efecto analgsico debido a fenmenos d& tolerancia y sobre todo a la progresin de la enfermedad. 3. La depresin respiratoria es un efecto secundario frecuente dada la disfuncin orgnica que presentan estos pacientes. 4. La morfina no tiene techo teraputico por lo que no existe una dosis mxima que no pueda sobrepasarse si lo requiere el paciente. 5. Se deben pautar a intervalos regulares y no slo si el paciente presenta dolor. 137. Un hombre de 84 aos ingresado en una residencia de ancianos est diagnosticado de Enfermedad de Alzheimer moderada. No toma medicamentos psicotrpicos y duerme bien. En ocasiones ha presentado ansiedad e ideas delirantes de intensidad leve, pero en las ltimas dos semanas ha pasado cada vez ms tiempo deambulando por los vestbulos. Varias veces ha entrado en las habitaciones de otros residentes y ha burgado en sus pertenencias. En una ocasin tambin se ha escapado de la Residencia. Cul de las siguientes estrategias de tratamiento es la ms apropiada?: 1. Proporcionarle actividad fsica estructurada y acompaarlo en sus paseos por el exterior. 2. Utilizar de forma intermitente un chaleco de restriccin en una silla durante el da. 3. Utilizar restricciones de muecas durante los episodios de deambulacin ms prolongados. 4. Prescribir risperidona, 1 mg por va oral, dos veces al da. 5. Trasladar al paciente a otro centro geritrico especializado 138. Hombre de 29 aos de edad que es trado a urgencias de un Centro de Salud por unos vecinos al habrselo encontrado inconsciente en la calle. A la exploracin destaca: pulso a 54 latidos por minuto, bradipnea con paradas respiratorias cada vez ms frecuentes y prolongadas, hipotermia, cianosis central y pupilas miticas. Cul de las siguientes actitudes teraputicas iniciales es la ms correcta?: 1. 2. 3. 4. 5. Asegurar la Asegurar la Asegurar la Asegurar la Asegurar la va area, oxgeno y suero glucosado 10%. va area, calentamiento corporal y observacin. va area, administrar dopamina intravenosa y observacin. va area, administrar naloxona intravenosa y observacin. va area, administrar naltrexona y observacin.

139. 1. 2. 3. 4. 5.

No representa un beneficio de la Ciruga sin ingreso:

Beneficios psicolgicos, en especial en nios. Menor infeccin de la herida. Un menor consumo de analgsicos. Reduccin de la lista de espera. Reduccin de los costes sanitarios.

140. Los opioides son frmacos de uso sistemtico durante la anestesia. En referencia a dichos frmacos. Cul de las siguientes afirmaciones es FALSA?: 1. Reducen la concentracin alveolar mnima (CAM) de los anestsicos inhalatorios. 2. Producen analgesia profunda y mnima depresin cardiaca.

3. Pueden producir hipnosis y amnesia. 4. Pueden producir bradipnea y depresin respiratoria. 5. Slo son eficaces por va intravenosa, 141. En qu gen deben buscarse mutaciones en un paciente joven afecto de un carcinoma medular de tiroides para determinar si existe un riesgo familiar a padecer una adenomatosis endocrina mltiples (MEA)?: 1. 2. 3. 4. 5. RET. APC. MSH2. c-KIT. c-MYC.

142. Las mutaciones del gen GJB1 situado en la regin cromosmica Xql3.1 son causa de una neuropata hereditaria denominada Charcot-Marie-Tooth tipo CMT1X. Un paciente de 25 aos diagnosticado de CMT1X (caso ndice) y al que le hemos hallado una mutacin validada clnicamente acude a la consulta de consejo gentico con sus dos hijos por el momento clnicamente sanos: un nio de 1 ao y una nia de 3. Cules familiares directos deben ser porta dores obligados de la enfermedad y en los que indicaremos pruebas genticas para estudiar su transmisin a la descendencia?: 1. 2. 3. 4. 5. Ambos hijos. El hijo de 1 ao. La nia de 3 aos. La esposa del caso ndice. La nia de 3 aos y la madre del caso ndice. Qu lesin de la mucosa oral debe ser considerada como precancerosa?:

143. 1. 2. 3. 4. 5.

Leucoplasia. Pnfigo. Enfermedad de Behet. Estomatitis aosa recidivante. Candidiasis oral.

144. Hombre de 69 aos con antecedentes de hipertensin arterial e hiperuricemia en tratamiento con amlodipino y alopurinol. Hace unas 2 se manas present un cuadro de lumbalgia por el que recibi tratamiento con diclofenaco. Acude a urgencias del hospital por presentar desde hace 48 horas una erupcin cutnea confluyente en tronco y extremidades. A la exploracin se observan lesiones maculares eritematoviolceas muy extensas (>70% de la superficie cutnea) sobre las cuales aparecen ampollas y erosiones. Existe afectacin (lesiones erosivo-costrosas) de mucosa labial, oral y conjuntiva. El signo de Nikolsky es positivo. El diagnstico probable es: 1. 2. 3. 4. 5. Penfigoide ampolloso. Necrolisis txica epidrmica. Pnfigo vulgar. Pnfigo paraneoplsico. Liquen ampolloso.

145. Un paciente de 22 aos acude a urgencias en shock anafilctico, posiblemente despus de tomar jarabe para la tos. No presenta, antecedentes personales de inters y a la exploracin se aprecian lesiones maculosas, marronceas no descamativas y generalizadas de 1-2 cm. de dimetro que en zonas de roce estn elevadas, ms eritematosas y pruriginosas. Al ser interrogado indica que han ido apareciendo y progresando en los diez ltimos aos. Qu enferme dad presenta este paciente que relacione todos los sntomas?: 1. 2. 3. 4. 5. Liquen plano atrfico. Amiloidosis. Psoriasis. Mastocitosis. Candidiasis mucocutnea crnica.

146. Cul de estos procesos puede producir simultneamente afectacin ocular y cutnea?: 1. 2. 3. 4. 5. Acn vulgaris, Lupus eritematoso discoide. Dermatitis herpetiforme. Roscea, Dermatomiositis.

147. Con relacin a las manifestaciones clnicas del liquen plano, todas las respuestas son correctas, EXCEPTO: 1. 2. 3. 4. 5. Distribucin simtrica, en zonas flexurales. Prurito en la mayora de los pacientes. La variante anular se localiza con preferencia en el pene. La variante hipertrfica se localiza preferentemente en el cuero cabelludo. La afectacin oral aparece en los 2/3 de la totalidad de los casos.

148. Un paciente de 48 aos, acude a urgencias aquejando un intenso dolor en el ojo izquierdo que le sobrevino de forma brusca mientras vea televisin. A la exploracin se evidencia que ese ojo izquierdo est enrojecido, con discreto edema corneal y la pupila se encuentra en midriasis media, con pobre respuesta a la luz. Ante el cuadro clnico descrito, qu opcin le parece MENOS probable de las siguientes propuestas?: 1. El cuadro clnico expuesto suele ser ms frecuente en personas con hipermetropa. 2. En esta situacin se espera que la presin infraocular de ese ojo est elevada. 3. Se trata de un cuadro ms frecuente en pacientes sin cristalino (afquicos). 4. Probablemente este paciente tendr una cmara anterior estrecha. 5. Es probable que este paciente refiera ver halos alrededor de las luces 149. Hombre de 66 aos que acude a consulta por dolorimiento y enrojecimiento del ojo derecho de 2 das de evolucin. La exploracin efectuada en ese ojo nos revela los siguientes datos: discreta disminucin de la agudeza visual, presin intraocular de 18 mmHg, pupila en miosis, presencia de clulas en humor acuoso, opacidad cristaliniana moderada y presencia de una sinequia iris-cristalino. Cul sera la actitud inmediata ms correcta de las que se enumeran a continuacin?: 1. 2. 3. 4. 5. Ciruga de la catarata de ese ojo, responsable del cuadro clnico. Tratamiento tpico (colirios) con midriticos y corticoides. Ciruga del glaucoma (trabeculectoma clsica o esclerectoma profunda no perforante). Manitol intravenoso asociado a tratamiento mitico y corticoides tpicos. Tratamiento con colirios antibiticos tpicos. La Uvetis por reconstitucin inmune es una entidad:

150. 1. 2. 3. 4. 5.

Asociada al HLA-B27 Descrita en pacientes VIH. Muy frecuente en pacientes con lupus eritematoso sistmico. Que ocurre en algunos pacientes con sfilis ocular tras la administracin de penicilina. Que ocurre en algunos pacientes con sarcoidosis tras abandonar el tratamiento con corticoides.

151. Mujer de 65 aos, hipertensa y diagnosticada de diabetes no insulinodependiente, con buen control metablico, acude a consulta por prdida importante de agudeza visual en su ojo derecho de 4 das de evolucin. En la exploracin del fondo de ojo destaca la presencia de venas dilatadas y tortuosas, hemorragias en llama, edema retiniano difuso y algunas manchas algodonosas; no se observan alteraciones significativas en cabeza de nervio ptico (papila). Con los datos indicados, seale el diagnstico ms probable: 1. 2. 3. 4. Oclusin de la arteria central de la retina. Obstruccin de la vena central de la retina. Retinopata diabtica proliferativa. Retinopata hipertensiva grado IV de Keith-Wegener.

5. Hemorragia vtrea. 152. 1. 2. 3. 4. 5. La angina de Ludwig:

Es una forma de amigdalitis abscesificada. Asocia edema de labio con parlisis facial y tumefaccin parotdea. Debe ser tratada exclusivamente con antiinflamatorios no esteroideos. Se produce habitualmente en pacientes inmunodeprimidos, siendo su etiologa mictica. Es una forma grave de infeccin que afecta al suelo de la boca y la regin submandibular.

153. Ante un paciente fumador de 45 aos que presenta disfona y mediante exploracin se observa una lesin vegetante rugosa en el borde libre de la cuerda vocal derecha cul es la conducta a seguir?: 1. 2. 3. 4. 5. Reposo vocal. Conducta expectante. Derivacin para biopsia. Supresin del tabaco. Braquiterapia. Cul es el tratamiento de eleccin del absceso periamigdalino?:

154. 1. 2. 3. 4. 5.

Amoxicilina clavulnico. Corticoesteroides. Clindamicina. Eritromicina. Quirrgico. El principal germen responsable de la otitis externa difusa es:

155. 1. 2. 3. 4. 5.

Pseudomona Aeruginosa Proteus. Candida Albicans. Haemophilus Influenzae. Staphylococcus Aureus. La hipoacusia caracterstica de la enfermedad de Menire es:

156. 1. 2. 3. 4. 5.

Hipoacusia de transmisin. Hipoacusia neurosensorial sin reclutamiento. Hipoacusia neurosensorial fluctuante. Hipoacusia mixta, de transmisin y neurosensorial. Hipoacusia neurosensorial retrolaberntica.

157. Uno de los siguientes sntomas caractersticos del sndrome esquizofrnico no puede considerarse un sntoma negativo: 1. 2. 3. 4. 5. Anhedonia. Alogia. Frialdad Emocional. Paralogia. Abulia.

158. Cul de los siguientes enunciados respecto a la depresin mayor o episodio depresivo mayor es INCORRECTO?: 1. A lo largo de la vida, aproximadamente un 15% de la poblacin sufre un episodio depresivo mayor. 2. Cerca de un 8% de los pacientes que atiende un mdico de Atencin Primara podra recibir un diagnstico de depresin mayor. 3. Hasta un 15% de los casos de depresin mayor son secundarios a una enfermedad mdica somtica o al abuso de substancias.

4. Cuando se sospecha que un paciente sufre un episodio depresivo mayor, debe evitarse hacer preguntas directas al paciente sobre ideas o proyectos suicidas, porque tal pregunta aumenta el riesgo de suicidio un 35%. 5. Cerca del 15% de los pacientes con depresin mayor que no reciben tratamiento se suicidan. 159. La diferencia fundamental entre "rasgo de personalidad" y "trastorno de personalidad" es la siguiente: 1. No existen diferencias entre ambos conceptos. Se utilizan de modo equivalente. 2. El rasgo de personalidad es un patrn persistente de funcionamiento cognitivo, afectivo y relacional inflexible, desadaptativo y causa malestar. 3. El trastorno de la personalidad es un patrn persistente de funcionamiento cognitivo, afectivo y relacional inflexible, desadaptativo y causa malestar. 4. El rasgo de personalidad es por definicin patolgico y como tal debe de tratarse. 5. El rasgo de personalidad conduce inevitablemente hacia un trastorno de la personalidad. 160. Sealar cul de las siguientes afirmaciones es FALSA en relacin a los efectos adversos de las sales de litio en el tratamiento del trastorno bipolar: 1. El litio puede ocasionar alteraciones de la funcin renal. 2. El litio puede causar alteraciones de la electrofisiologa cardiaca. 3. El litio puede provocar exacerbacin de la psoriasis. 4. El litio puede provocar insuficiencia respiratoria. 5. El litio puede producir hipotiroidismo. 161. Mujer de 47 aos sin antecedentes psiquitricos que es hospitalizada para estudio de metrorragias y a los 5 das de su ingreso es informada del diagnstico de neoplasia uterina con metstasis. Se consulta a Psiquiatra porque 24 horas despus muestra tristeza y llanto frecuente, refiere ideas de muerte y presenta insomnio. En la evaluacin psiquitrica no se recoge ningn antecedente psiquitrico y la exploracin detecta elevada ansiedad y desesperanza en relacin con las consecuencias de su enfermedad neoplsica. El diagnstico ms probable es: 1. 2. 3. 4. 5. Trastorno esquizoafectivo. Distimia. Trastorno de ansiedad generalizada. Trastorno por estrs postraumtico. Reaccin normal frente al estrs.

162. Sealar el trastorno de la personalidad en el que es ms frecuente la presencia de autolesiones: 1. 2. 3. 4. 5. Trastorno narcisista de la personalidad. Trastorno paranoide de la personalidad. Trastorno antisocial de la personalidad. Trastorno lmite de la personalidad. Trastorno obsesivo de la personalidad. La caracterstica ms especfica del sndrome esquizofrnico es:

163. 1. 2. 3. 4. 5.

La existencia de trastornos de la percepcin. La incapacidad para distinguir entre la realidad interior y la exterior. El marcado simbolismo que expresa su conducta. La hipersensibilidad. La incongruencia del pensamiento.

164. Un seor de 60 aos, empleado de comercio, acude a urgencias acompaado de su familia; stos cuentan que el da anterior, de forma bastante brusca, comenz a decir cosas raras, a no responder a lo que le preguntaban y a mostrar se confuso incluso respecto de su propio nombre. Efectivamente en la exploracin parece no entender lo que se le pregunta, no recuerda nada de lo que le ha pasado y no sabe ni el da, ni el lugar en el que est. El diagnstico sera: 1. Psicosis breve. 2. Delirium.

3. Alzheimer. 4. Esquizofrenia. 5. Trastorno de ansiedad. 165. Un paciente bebedor habitual de alcohol es llevado a urgencias con un cuadro de desorientacin, dificultad para recordar hechos de los das anteriores, errores en los razonamientos, marcha torpe y una desviacin en los ojos divergente que antes no tena. Segn parece, ha estado bebiendo alcohol hasta unas horas antes de llevarlo a urgencias. Probablemente se trate de un caso de: 1. 2. 3. 4. 5. Sndrome de abstinencia complicado. Enfermedad de Korsakoff. Encefalopata de Wernicke. Alucinosis alcohlica. Intoxicacin etlica aguda.

166. En cul de los siguientes casos la terapia electroconvulsivante es un tratamiento de primera eleccin?: 1. 2. 3. 4. 5. Esquizofrenia paranoide. Psicosis reactiva. Depresin mayor con sntomas melanclicos. Trastorno esquizotpico de la personalidad. Trastorno delirante crnico.

167. El sndrome de ovario poliqustico es una enfermedad bien conocida de la mujer en edad frtil y su diagnstico se basa en los datos que aporta: 1. Historia clnica familiar pormenorizada ya que lo ms importante son los antecedentes familiares. 2. Realizacin de una Resonancia Nuclear magntica de la hipfisis para descartar un adenoma. 3. Estudio ecogrfico ginecolgico con tecnologa tridimensional y biopsia. 4. Valoracin clnica, estudio hormonal (andrgenos, FSH y LH) y ecografa ginecolgica. 5. Un escner para estudiar de forma resolutiva los ovarios y estudio hormonal completo. 168. Una mujer de 18 aos consulta por amenorrea primaria. En la exploracin se observa un fenotipo femenino normal pero con ausencia de desarrollo de los caracteres sexuales secundarios. Los genitales externos son femeninos de aspecto infantil y los genitales internos femeninos e hipoplsicos. La talla es normal. El cariotipo es 46 XX. Los niveles plasmticos de gonadotropinas (FSH y LH) estn muy elevados, Cul de los siguientes diagnsticos corresponde a este cuadro?: 1. 2. 3. 4. 5. Sndrome de insensibilidad a los andrgenos (feminizacin testicular). Disgenesia gonadal tipo Sndrome de Turner. Disgenesia gonadal pura. Defecto congnito de hormona liberadora de gonadotropinas (Gn-RH). Sndrome adrenogenital.

169. Mujer de 60 aos. En la revisin rutinaria se detecta una masa mvil en pelvis. El estudio ecogrfico pone de manifiesto, como nico hallazgo, una lesin qustica de 70 mm con proliferaciones papilares internas y dependiente de ovario derecho. El marcador tumoral Cal25 es de 70 U/mL (normal < 35 U/ml). Cul es la conducta diagnstico/teraputica adecuada?: 1. Control evolutivo mediante seriacin ecogrfica y del nivel del marcador. 2. Completar el estudio mediante marcadores de estirpe germinal (alfafetoprotena/gonadotropina corinica). 3. Estudio citolgico de la lesin mediante puncin aspiracin con aguja fina. 4. Tratamiento quirrgico: anexectoma y estudio intraoperatorio de la lesin. 5. Tratamiento quirrgico: quistectoma y estudio intraoperatorio de la lesin. 170. Cul es el tratamiento correcto para una pareja estril por una obstruccin bilateral de las trompas?:

1. 2. 3. 4. 5.

La insuflacin de las trompas. La inseminacin artificial con semen del marido. La inseminacin artificial con semen de donante. Inducir la ovulacin con gonadotropinas. La fecundacin in vitro.

171. Cul de las siguientes situaciones clnicas es una contraindicacin absoluta para anticoncepcin hormonal?: 1. Hipertensin arterial bien controlada. 2. Historia personal de tromboembolismo previo. 3. Diabetes. 4. Mujer de menos de 35 aos y fumadora. 5. Infeccin urinaria. 172. Los factores pronsticos condicionan la evolucin en el cncer de cuello uterino. Cul de stos NO sera importante para el estadiaje clnico del tumor?: 1. 2. 3. 4. 5. Tamao tumoral. Afectacin ganglionar. Profundidad de invasin tumoral. Permeabilidad vascular. Vaginitis de repeticin.

173. A una gestante a trmino que ingresa en trabajo de parto se le detectan unas pequeas vesculas vulvares de herpes simple recidivante. Aos antes de la gestacin tuvo una primoinfeccin de herpes genital y varios brotes de herpes recidivante. Cul es la conducta a seguir?: 1. Hacer una cesrea. 2. Permitir el parto vaginal y tratar con aciclovir al recin nacido. 3. No es necesario una conducta especial ya que el herpes recidivante no tiene riesgo para el recin nacido. 4. Permitir el parto vaginal y aislar al recin nacido de la madre. 5. Tratar inmediatamente las lesiones con cido tricloroactico para inactivar el virus y entonces permitir el parto vaginal. 174. Cul de los siguientes factores NO es un factor de riesgo para desprendimiento prematuro de placenta normalmente inserta?: 1. 2. 3. 4. 5. Hipertensin arterial crnica. Nuliparidad, Tabaquismo. Hipofibrinogenemia congnita. Presencia del anticoagulante lpico en sangre.

175. Cul de las siguientes situaciones NO supone una indicacin primaria de la ecografa obsttrica durante el primer trimestre de gestacin?: 1. 2. 3. 4. 5. Definir la causa de un sangrado vaginal. Confirmar la edad gestacional. Valorar una sospecha de enfermedad trofoblstica gestacional. Diagnosticar un embarazo en mujer con amenorrea. Evaluar un dolor plvico agudo.

176. En una mujer embarazada de 11 semanas que lleva tres das sangrando por genitales, con muchas nuseas, tero mayor que su amenorrea y unos valores de beta HCG muy elevados, en cul de las siguientes patologas debe pensarse?: 1. 2. 3. 4. 5. Amenaza de aborto. Aborto diferido. Mola hidatdica. Amenaza de aborto en un tero con miomas. Aborto incompleto.

177. Cul de las siguientes serologas es ms recomendable hacer a todas las embarazadas?: 1. 2. 3. 4. 5. La del citomegalovirus. La del parvovirus. La de la hepatitis C. La del SIDA. La de la listeriosis.

178. Seala cul de las siguientes afirmaciones sobre los genes BRCA1 y BRCA2 implicados en el cncer de mama hereditario es FALSA: 1. El riesgo de padecer cncer de mama en mujeres portadoras de mutaciones en alguno de estos genes es del 60-85%. 2. El riesgo de padecer adems cncer de ovario es mayor en las mujeres portadoras de mutacin en el BRCA1 que en las portadoras de mutacin en el BRCA2. 3. Los cnceres de mama asociados a mutaciones en el BRCA1 son generalmente bien diferenciados en comparacin con los cnceres de mama espordicos. 4. Mutaciones en los genes BRCA1 y BRCA2 son muy poco frecuentes en cnceres de mama espordicos. 5. La prevalencia de mutaciones del BRCA1 es mayor que la del BRCA2. 179. Paciente de 51 aos que en mamografa de cribado se detecta ndulo espiculado de 8 mm en cuadrante superoexterno de la mama izquierda. Biopsia con agua gruesa: carcinoma ductal infiltrante G IL Receptores hormonales positivos. Ki 67 10%, Her 2-neu negativo. Exploracin clnica: ndulo no palpable, axila libre, Ecografa axilar: No ganglios sospechosos. Cul es el tratamiento primario de eleccin?: 1. 2. 3. 4. 5. Hormonoterapia primaria. Quimioterapia neoadyuvante. Mastectoma y linfadenectoma axilar. Tumorectoma y ganglio centinela. Trastuzumab.

180. Paciente de 4 aos que acude a urgencias con una historia de secrecin nasal purulenta y de mal olor unilateral desde hace 4 das. El dia gnstico ms frecuente es: 1. 2. 3. 4. 5. Fibrosarcoma del cornete nasal. Atresia de coanas unilateral. Rinitis crnica por Rinovirus. Cuerpo extrao intranasal. Granulomatosis crnica nasofarngea.

181. En relacin con el crecimiento y desarrollo del nio en su primer ao de vida, todas las afirmaciones siguientes son ciertas, EXCEPTO: 1. En el primer ao de vida, el nio aproximadamente triplica su peso al crecimiento y duplica su longitud. 2. El peso del recin nacido suele disminuir ms del 10% en la primera semana de vida, por prdida de lquido extravascular, aunque se recupere posteriormente. 3. Un nio debe ganar alrededor de 30 gramos/da durante el primer mes de vida. 4. Los reflejos precoces o arcaicos presentes al nacimiento, desaparecen entre los 3 y 4 meses de edad. 5. Hacia los 2 meses, suele haber hecho su aparicin la sonrisa voluntaria o social. 182. Paciente de 3 meses con linfopenia, neumona por Pneumocystis Carinii junto con muguet de repeticin e incapacidad para ganar peso. El diagnstico ms probable es: 1. 2. 3. 4. 5. Deficiencia de IgA. Inmunodeficiencia combinada grave. Sndrome variable comn de inmunodeficiencia. Enfermedad granulomatosa crnica. Deficiencia en molculas de adhesin

183. Un nio con anorexia, fatigabilidad e irritable, adoptando una postura con las extremidades semiflexionadas y abducidas, que Hora al movilizarlo y tiene gingivitis hemorrgica e hiperqueratosis folicular, tendr un dficit de: 1. 2. 3. 4. 5. Vitamina A. Triptfano. Vitamina C. Selenio. Vitamina K.

184. Nia de 2 aos que estando previamente bien sufre episodio brusco de desconexin del medio e hipertona, con estridor y sialorrea, durante aproximadamente 2 minutos, quedando posteriormente somnolienta durante aproximada mente 5 minutos. A su llegada al Centro de Salud se objetiva temperatura axilar de 38,9C. En relacin con esta nia, cul de las siguientes afirmaciones es correcta?: 1. Presenta un proceso benigno pero que casi siempre recurre. 2. Debe remitirse a un Centro Hospitalario para realizacin de una prueba de neuroimagen. 3. Si el foco causal de la fiebre es claro y banal y la recuperacin clnica completa, puede manejarse con antitrmicos y observacin domiciliaria sin realizar ninguna prueba complementaria. 4. Ser necesaria la realizacin de un EEG ms adelante. 5. Presenta un riesgo de desarrollo posterior de alguna forma de epilepsia mayor que el resto de la poblacin. 185. Se encuentra ante un recin nacido a cuya madre se le descubri una seroconversin a Toxoplasmosis durante la gestacin. Cul de las siguientes afirmaciones es FALSA?: 1. 2. 3. 4. 5. El estudio de la placenta puede resultar de utilidad en el diagnstico. Debe realizar cuanto antes un estudio serolgico al recin nacido. El cuadro clnico neonatal puede ser indistinguible de otras infecciones congnitas. El recin nacido puede encontrarse asintomtico. Los recin nacidos infectados pero no enfermos no precisan tratamiento.

186. Lactante de 3 meses de edad que acude a la consulta por presentar desde los 15 das de vida, regurgitaciones postprandiales en casi todas las tomas y de forma espordica, vmitos en mayor cantidad. Heces de caractersticas normales. Lactancia artificial con frmula de inicio y presenta un adecuado desarrollo ponderoestatural, sin prdida de peso. Cul entre los siguientes, es el diagnstico ms probable?: 1. 2. 3. 4. 5. Alergia a protena de leche de vaca. Estenosis pilrica. Intolerancia a protenas de leche de vaca. Reflujo gastroesofgico madurativo. Vlvulo intestinal.

187. En un nio de 11 aos con asma persistente moderada, y sensibilizacin a caros de polvo domstico y plenes de ariznicas, gramneas y olea, todas las siguientes medidas son recomen dables EXCEPTO una: 1. Vacunar anualmente contra la gripe. 2. Tratar la rinitis y la sinusitis coincidentes. 3. Ensearle a medir el flujo espiratorio mximo en su domicilio. 4. Evitar o reducir la exposicin a los alrgenos a los que est sensibilizado y a otros irritantes de la va respiratoria. 5. Emplear como tratamiento de mantenimiento beta-agonistas de accin larga en monoterapia. 188. Una de las siguientes caractersticas clnicas NO es tpica de la intolerancia a disacridos: 1. Diarrea postprandial. 2. Deposiciones de olor cido. 3. Eritema perianal.

4. Deposiciones explosivas. 5. Heces abundantes, brillantes y adherentes (esteatorreicas). 189. Cuando se diagnostica de fimosis a un lactante, al comentarles sobre el proceso a sus padres tenemos que tener en cuenta que: 1. 2. 3. 4. 5. Hasta los dos aos se puede tratar de un proceso fisiolgico. Es un hallazgo poco frecuente. Se realiza circuncisin solo en caso de existir motivos religiosos. Las adherencias peneanas y la fimosis se tratan de una misma patologa. En la actualidad el uso de pomadas de corticoides evita la circuncisin por motivos mdicos.

190. Entre otras manifestaciones, la talla baja est presente en mltiples sndromes genticos y polimalformativos, como los que se exponen a continuacin, EXCEPTO en uno. Cul de ellos NO suele cursar con talla baja?: 1. 2. 3. 4. 5. Sndrome de Turner. Sndrome de Down. Sndrome de Silver-Russell. Sndrome de Seckel Sndrome de Klinefelter.

191. Recin nacido prematuro de 3 horas de vida que presenta cianosis persistente, con hipoxemia, que mejoran escasamente con 02 al 100%. La radiografa de trax muestra pulmones hiperclaros, con corazn en bota. Seale la respuesta correcta: 1. El nio presenta una enfermedad de la membrana hialina. 2. Debe instaurarse una terapia intravenosa con prostaglandina E para mantener permeable el conducto arterioso. 3. La ecocardiografa bidimensional no ser diagnstica en este caso. 4. El tratamiento de eleccin inicial es la ventilacin mecnica. 5. El paciente rene todos los criterios de insuficiencia cardiaca neonatal. 192. Si se realiza el diagnstico de osteocondritis de la epfisis femoral superior o enfermedad de Legg-Calv-Perthes es verdad que: 1. La edad de aparicin es en mayores de 10 aos. 2. Aparece especialmente en el sexo femenino. 3. La cojera es el sntoma predominante con mayor o menor dolor. 4. En la exploracin del nio llama la atencin la importante afectacin del estado general. 5. La evolucin de la enfermedad es rpida, siendo a evolucin ms frecuente de aproximadamente un mes. 193. En un estudio clnico, a una serie de pacientes se les trata con un nuevo frmaco para estudiar si, en un periodo de tiempo despus de la administracin de dicho frmaco, el nivel de bilirrubina ha disminuido. Se acepta que la distribucin de la bilirrubina es normal, para este diseo. Cul es la prueba estadstica de eleccin?: 1. T de Student para datos apareados. 2. T de Student para datos independientes. 3. Chi-cuadrado. 4. Mann-Whitney. 5. Prueba exacta de Fisher 194. Supongamos un modelo de regresin lineal simple que relaciona los niveles medios de Protena C Reactiva (PCR) medida en mmol/L como variable dependiente con el nmero de articulaciones dolorosas o hipersensibles de un paciente con artritis idioptica (variable independiente). El modelo ajustado en una muestra de 100 pacientes es el siguiente: fjy = 0,47 + 0,08* En el modelo. Cmo se interpreta el coeficiente de 0,08?: 1. Dado que es superior a 0,05, quiere decir que no existe asociacin entre las variables dependiente e independiente. 2. Es el cambio obtenido en la media de PCR como consecuencia de pasar a tener una articulacin adicional inflamada.

3. Es el valor medio de la PCR para los sujetos con una sola articulacin afecta. 4. Su cuadrado se interpreta como el porcentaje de varianza de la variable dependiente explicado por el modelo lineal. 5. Es el valor medio basal de PCR para los individuos sanos (sin articulaciones afectas). 195. En un contraste de hiptesis estadstico, si la hiptesis nula fuera cierta y se rechazara: 1. 2. 3. 4. 5. Se comete un error de tipo II. Se toma una decisin correcta. La potencia aumenta. Se comete un error de tipo I. Se toma la decisin ms conservadora.

196. En un ensayo clnico para comparar dos frmacos, se incluyen 75 pacientes en cada rama, la variable de inters es tiempo hasta que desaparecen los sntomas y el perodo de seguimiento es de 30 das. Al final de ese perodo hay pacientes a los que no les han desaparecido los sntomas. El anlisis de los datos: 1. Se puede hacer con la prueba t de Student. 2. Se debe hacer con la prueba ji-cuadrado. 3. Se debe hacer con las tcnicas de anlisis de supervivencia. 4. Se debe hacer con la prueba de Mann-Withney. 5. Se puede hacer con la prueba de Mann-Withney, o con las tcnicas de anlisis de supervivencia. 197. Un ensayo clnico sobre 3.000 pacientes compara un nuevo frmaco antihipertensivo con placebo (1.500 pacientes por rama de tratamiento), Para cada grupo, se obtiene un valor medio de tensin arterial. Para comprobar si existen diferencias estadsticamente significativas, se debe realizar la siguiente prueba: 1. 2. 3. 4. 5. La X2 de Pearson. La t de Student. La F de Snedecor. La U de Marm-Whitney. Hazard Ratio.

198. Se va a realizar un estudio para comprobar si la administracin de determinado antipsictico podra estar relacionado con un aumento de la incidencia de conductas suicidas en esquizofrnicos en los que se inicia el tratamiento de un brote agudo. Los pacientes ms graves pueden ser ms proclives a serles indicado este antipsictico y tambin tienen mayor riesgo suicida. Teniendo en cuenta este posible factor de confusin "por indicacin'*. Cul de los siguientes tipos de estudio minimiza mejor en el diseo la posibilidad de que se produzca este sesgo de confusin?: 1. 2. 3. 4. 5. Un ensayo clnico aleatorizado. Un estudio de casos y controles apareado. Un estudio de cohortes de seguimiento prospectivo, Un estudio de prevalencia. Un estudio de casos y controles anidado en una cohorte. Respecto a los estudios epidemiolgicos de cohorte es cierto que:

199.

1. Se utiliza la Odds Ratio como medida fundamental de asociacin. 2. Se denominan tambin estudios de prevalencia. 3. El grupo de comparacin se constituye con los sujetos no expuestos al factor de riesgo en estudio. 4. Estudian las caractersticas de una muestra de sujetos en un momento dado, en el que se recogen todos los datos. 5. Si existen prdidas en el seguimiento de los sujetos, el estudio no puede analizarse. 200. Cul de los siguientes estudios se corresponde con el diseo de un estudio de casos y controles?:

1. Seleccionamos un grupo de pacientes diagnosticados de agranulocitosis y un grupo control de pacientes del mismo hospital con otras enfermedades hematolgicas. Comparamos a los dos aos en ambos grupos la proporcin de pacientes que sobreviven. 2. Seleccionamos un hospital en el que han ocurrido una serie de casos de legionelosis y como control, otro hospital sin ningn caso. Comparamos la temperatura del agua en los depsitos de agua caliente de cada hospital. 3. Seleccionamos un grupo de diabticos e hipertensos en tratamiento con enalapril y otro grupo con las mismas enfermedades en tratamiento con diurticos. Comparamos en ambos grupos los valores de proteinuria despus de 6 meses de tratamiento. 4. Seleccionamos una muestra de sujetos que han tenido un resultado positivo en un control de alcoholemia mientras conducan. Calculamos en este grupo la prevalencia de cardiopata isqumica sintomtica. 5. Seleccionamos un grupo de pacientes diagnosticados de infarto agudo de miocardio y un grupo seleccionado de forma aleatoria procedente de la misma colectividad, sin infarto. Comparamos en ambos grupos la proporcin de pacientes previamente expuestos a determinado anti-inflamatorio no esteroideo. EXAMEN 2009

Actualmente, en el Programa de Inmunizaciones del MINSA NO est incluida la vacuna contra: A. Difteria B. Sarampin C. Poliomielitis D. Varicela E. Hepatitis B El nivel ptimo de Participacin Comunitaria en Salud, considera que la comunidad participa en: A. Evaluacin de actividades B. Apoyo econmico C. Apoyo de materiales D. Ejecucin de actividades E. Toma de decisiones En la colecistitis aguda, Cul es el tratamiento quirrgico de eleccin?: A. Colecistostoma B. Colecistectoma convencional C. Colecistectoma laparoscpica D. Colecistectoma diferida E. Coledocoduodenoanastomosis La presencia de sangre en la cmara anterior del ojo se denomina: A. Hipopion B. Hifema C. Hipertropia D. Hiperforia E. Hiperopia Paciente gestante de 38 aos. G:5 P:4, con 37 semanas de edad gestacional, portadora de un mioma subseroso de 9 cm. en cara anterior e inferior del cuerpo uterino, sin molestias presentes. Cul es la conducta a seguir?: A. Operacin cesrea corporal B. Operacin cesrea segmentaria C. Control de la paciente D. Operacin cesrea segmentaria seguida de miomectoma E. Operacin cesrea corporal seguida de miomectoma La principal complicacin de la pancreatitis aguda es: A. Diabetes mellitus B. Pseudoquiste pancretico C. Desnutricin D. Ictericia

E. Esteatorrea El tumor maligno de estmago ms frecuente es: A. Adenocarcinoma B. Carcinoma adenoescamoso C. Carcinoma epidermoide D. Linfoma E. Leiomioma La causa ms frecuente de amenorrea es: A. Deficiencia de la hormona folculo estimulante B. Estrs C. Deficiencia de la hormona luteinizante D. Embarazo E. Enfermedad crnica debilitante En un paciente operado de cadera, obeso, con aparicin sbita de disnea, dolor torcico, tos, esputo hemoptoico; se debe sospechar: A. Pericarditis aguda B. Aneurisma disecante C. Pleuresa con derrame D. Neumona lobar E. Embolismo pulmonar En caso de meningoencefalitis bacteriana peditrica tratada eficazmente, el LCR se esterilizar en un 90% de los casos en: A. 48 a 72 horas B. menos de 24 horas C. 24 a 36 horas D. 4 das E. 1 semana Son factores de riesgo a considerar en el cncer de endometrio en pacientes postmenopusicas, EXCEPTO: A. Sangrado postmenopusico B. Dolor plvico C. Diabetes Mellitas D. Hipertensin Arterial E. Obesidad En el Trastorno Obsesivo Compulsivo, el patrn sintomtico ms comn es la obsesin: A. De contenido sexual y la compulsin al castigo B. De la duda seguida de la compulsin de comprobar C. Del miedo a la contaminacin seguida de la compulsin de lavados D. De simetra y precisin seguida de lentitud compulsiva E. Del miedo a pecar seguida de la compulsin de rezar En un paciente con insuficiencia renal crnica Cul de las alteraciones en el ECG NO tiene relacin con el diagnstico de hiperkalemia? A. Segmento P-R aumentado B. Onda T isoelctrica C. Onda P disminuida de amplitud D. Onda U E. QRS ensanchado En el alumbramiento de un parto normal, el desprendimiento de la placenta se produce fundamentalmente por: A. Compresin del tero por el mdico B. Formacin del hematoma retroplacentario C. Disminucin rpida de progesterona D. Presin abdominal por esfuerzos matemos (pujos) E. Contracciones uterinas y retraccin del tero La patologa que NO produce insuficiencia cardiaca congestiva es: A. Beri beri B. Tirotoxicosis C. Artritis reumatoidea D. Anemia crnica E. Vasculopatas pulmonares De las entidades citadas, Cul es la causa ms comn de quilotrax? A. Carcinoma B. Linfoma

C. Yatrogenia quirrgica D. Traumatismo torcico E. Congnita El mecanismo ms frecuente de infeccin bacteriana por cnulas intravenosas es: A. Siembra a partir de sitios lejanos debido a bacteriemia intermitente B. Contaminacin de lquidos durante el proceso de manufactura C. Contaminacin de lquidos durante la insercin de la cnula D. Contaminacin durante la inyeccin de medicamentos E. Contaminacin en el. sitio de entrada a travs de la piel Cul es el examen auxiliar ms til para hacer el diagnstico de monoartritis tuberculosa de rodilla? A. Biopsia de membrana sinovial B. Radiografa de trax C. Estudio del lquido sinovial D. Radiografa de rodilla en dos posiciones E. Reaccin de Mantoux Cul es el trastorno neurolgico ms frecuente que causa hipertermia? A. Lesin del ncleo supraquiasmtico B. Lesin del hipotlamo posterior C. Lesin del hipotlamo anterior D. Tumores en el tlamo E. Adenoma de hipfisis El Pie equino puede ser resultado de una lesin del nervio: A. Femoral B. Tibial posterior C. Tibial anterior D. Crural E. Pudendo El tercer grado de hemorroides internas, se caracteriza por: A. Prolapso con reduccin espontnea B. Hemorroides sin prolapso C. Prolapso que requiere reduccin digital D. Prolapso que no se puede reducir E. Prolapso y estrangulamiento La amenaza de parto prematuro se presenta en una edad gestacional cuyo rango es: A. Entre 20 y 24 semanas B. Entre 28 y 37 semanas C. Menos de 20 semanas D. Entre 38 y 40 semanas E. Mayor de 40 semanas La medicacin que permite una mejora rpida de un lactante con crup infeccioso grave es: A. Adrenalina en nebulizacin B. Dexametasona va endovenosa C. Ambiente hmedo D. Antibiticos E Salbutamol en inhalacin Son caractersticas del agente comunitario, EXCEPTO: A. Ser promotores de salud B. Ser reconocidos por la comunidad C. Ejercer en funcin de las necesidades de la poblacin D. Ser designados por el establecimiento de salud E. Desarrollar actividades de prevencin En la seleccin de sujetos en los estudios de casos y controles: A. No debe influir la exposicin o no exposicin al factor de riesgo B. Debe tenerse en cuenta la exposicin o no exposicin al factor de riesgo C. Los controles deben ser de una poblacin distinta a la de los casos D. El estado de exposicin debe determinarse antes de la enfermedad E. La proporcin de controles debe ser mayor que la de los casos En el asma agudo, el frmaco utilizado en el tratamiento inicial es; A. Metilxantinas

B. Corticosteroides C. &#946;2 agonistas de accin corta D. &#946;2 agonistas de accin larga E. Anticolinrgicos En un nio de 18 meses que llega con flexin dolorosa del codo despus de haber sido alzado de ese mismo brazo, el diagnstico es: A. Fractura cubital B. Luxacin del hombro C. Desgarro muscular D. Subluxacin de la cabeza del radio E. Fractura de Colles La droga antituberculosa que mejor acta en las lesiones caseosas es: A. Gentamicina B. INH C. Ciprofloxacino D. Levofloxacino E. Rifampicina Paciente gestante con diagnstico de SIDA y sin tratamiento previo, llega a la emergencia en periodo expulsivo. El tratamiento retroviral elegido debe ser: A. Zidovudina en la madre y en el neonato B. Lamivudina en la madre C. Zidovudina y Lamivudina en la madre D. Zidovudina en la madre E. Lamivudina en el neonato En la endometritis puerperal, el principal hallazgo es: A. Mamas turgentes B. Dolor plvico C. Fiebre persistente D. Loquios hemticos E. Polaquiuria Se define Infectividad como la capacidad del germen para que: A. Produzca enfermedad infecciosa B. Produzca reaccin inmune C. Ingrese y se multiplique en el hospedero D. Desarrolle enfermedad infecciosa severa E. Se transmita al hospedero El desprendimiento placentario es una complicacin de ______________ y la complicacin ms grave es ________________. A. Preeclampsia / muerte fetal B. Parto pretrmino / bito fetal C. Gestante aosa / hipertona uterina D. Anemia leve / hipofibrinogenemia E. Parto post trmino / hipocalcemia El embarazo gemelar que se produce en el primer da de la divisin celular del huevo, tiene las siguientes caractersticas: A. Monoplacentario y biamnitico B. Monoplacentario y monocorinico C. Monocorinico y monoamnitico D. Monocorinico y biamnitico E. Bicorinico y biamnitico La gingivoestomatitis herptica es la forma clnica ms frecuente de la infeccin primaria debido a: A. Herpes virus hominis tipo 1 B. Herpes virus hominis tipo 2 C. Herpes virus hominis tipo 6 D. Herpes virus varicela E. Virus Coxackie grupo A Se define Insuficiencia Renal Aguda como la disminucin ____________ de la funcin renal cuya consecuencia es _________________. A. Progresiva / hipernatremia B. Sbita / alteracin hdrica C. Sbita / hipokalemia D. Progresiva / hiperkalemia E. Progresiva / aumento de urea

El tratamiento de eleccin de las hemorroides externas trombosadas es: A. Dieta rica en fibra B. Ablacin quirrgica C. Baos de asiento D. Ligadura con banda de caucho E. Reduccin manual Paciente varn de 30 aos, sufre un accidente de trnsito. Presenta hematocrito de 20%, en shock hipovolmico Qu fluido debe administrarse?: A. Coloides B. Cristaloides C. Sangre total D. Paquete globular E. Plasma fresco En un paciente de 8 aos que acude a Emergencia con crisis asmtica, Cul de los siguientes signos indica que es severa?: A. Aleteo nasal B. Taquipnea C. Tirajes intercostales D. Cianosis E. Sibilancias en dos tiempos Principal examen auxiliar en el diagnstico de enfermedades micticas: A. Tincin de Gram B. Coloracin de Zhiel Nielsen C. Examen directo con hidrxido de potasio D. Tincin de Giemsa E. Tincin de Wright .Criterio para clasificar un cuadro clnico como Asma persistente moderada: A. Sntomas nocturnos mayor a una vez a la semana' B. Flujo espiratorio mximo menor o igual a 60% C. Sntomas nocturnos mayor a dos veces a la semana D. Crisis que pueden afectar la actividad E. Sntomas continuos La manifestacin clnica ms frecuente de la mola hidatiforme es: A. Dolor plvico B. Altura uterina mayor que edad gestacional C. Ausencia de actividad fetal D. Dolor torcico E. Sangrado vaginal De las siguientes articulaciones, las ms frecuentemente afectadas por la Artritis Reumatoide son: A. Hombros B. Interfalngicas dstales C. Metatarsofalngicas D. Tobillos E. Metacarpofalngicas En un nio de 3 aos de edad con intoxicacin por salicilatos, el primer signo es: A. Diplopa B. Petequias y sangrado gingival C. Hiperventilacin D. Diarrea y vmitos E. Convulsiones El diurtico de eleccin para el tratamiento de ascitis en pacientes cirrticos es: A. Hidroclorotiazida B. Espironolactona C. Furosemida D. Acetazolamida E. cido etacrnico Cul de las siguientes condiciones NO es causa de eritrodermia generalizada?: A. Reaccin adversa a medicamentos B. Psoriasis

C. Micosis fungoide D. Dermatitis exfoliativa E. Lupus discoide Cul es la edad ideal para operar el testculo no descendido?: A. Durante la pubertad B. A los cinco aos C. A los ocho aos D. A los 15 meses E. A los tres meses La causa ms comn de formacin de bulas en pulmn de nios con neumona es: A. La debilidad del intersticio pulmonar B. La capacidad necrotizante del estafilococo C. La virulencia del estreptococo D. La gran capacidad reproductiva de Klebsiella E. La intensidad de la tos El tratamiento antibitico que se recomienda en pacientes portadores de osteomielitis debe durar un mnimo de semanas: A. 3 B. 2 C. 10 D. 4 E. 8 El principal objetivo de los procedimientos quirrgicos en el manejo de la lcera pptica es: A. Reducir el tamao gstrico B. Reducir la produccin cida del estmago C. Facilitar el drenaje gstrico D. Reducir posibilidad de infeccin de Helicobacter pylori E. Tratar el dolor Cul es el mejor estudio para diagnosticar el traumatismo renal?: A. Tomografa rodal computarizada B. Urografa excretoria C. Ecografa renal D. Arteriografa renal E. Placa simple de abdomen El tratamiento definitivo para el quiste hidatdico pulmonar es: A. Broncoscopa B. Quistectoma C. Puncin transparietal D. Mebendazol E. Drenaje La vmica es caracterstica de: A. Absceso pulmonar B. Bronquiectasia C. Quiste hidatdico D. Tuberculosis pulmonar cavitada E. Aspergiloma pulmonar La Psoriasis inversa, afecta principalmente: A. Dorso de la mano B. Codos C. Rodillas D. Axilas E. Cuello La Enfermedad Diarreica Aguda caracterizada por transporte activo de solutos a la luz intestinal, es la diarrea: A. Por invasin de la mucosa B. Osmtica C. Secretora D. Por aumento de la motilidad E. Por mala absorcin La lesin conocida como ganglin es dependiente de: A. Vasos linfticos B. Clulas gigantes

C. Vasos sanguneos D. Cartlago E. Vaina sinovial En la atencin de un parto vaginal con presentacin plvica, existe el riesgo de lesiones en el sistema ___________ del recin nacido. A. Musculoesqueltico B. Respiratorio C. Nervioso central D. Cardiovascular E. Endocrino El examen auxiliar para realizar el diagnstico precoz de la osteomielitis es: A. Resonancia magntica B. Radiografa simple C. Tomografa axial D. Gammagrafa E. Velocidad de sedimentacin En relacin con la infeccin por el virus de VIH, se considera pareja discordante cuando uno de ellos: A. Es VIH seronegativo B. No tiene tratamiento anti-retroviral C. Tiene baja cantidad de linfocitos CD4 D. Tiene tiempo de enfermedad mayor E. Tiene carga virar diferente El examen ms til para el diagnstico de torsin testicular es: A. Urografa excretoria B. TAC C. Ecografa D. Eco Doppler E. Deferentografa En el manejo de una intoxicacin por fenotiaznicos, el antdoto es: A. Fenobarbital B. Biperideno C. Diazepam D. Orfenadrina E. Clonazepam Una cada sobre el dorso de la mano puede producir una fractura de: A. Articulacin carpo metacarpiana B. Colles C. Cabeza de radio D. Smith E. Falange proximal Para comparar la distribucin de la frecuencia de la edad de dos grupos se utiliza: A. Grfico de barras B. Grficos circulares C. Polgono de frecuencias D. Tallos y hojas E. Pictogramas Cul de las maniobras de Leopold sirve para determinar la presentacin del feto?: A. Cuarta B. Primera C. Segunda D. Tercera E. Quinta En el puerperio, la aparicin de una tumoracin con signos de flogosis en la mama, sugiere el diagnstico de: A. Mastitis B. Enfermedad de Paget C. Carcinoma ductal D. Papiloma intraductal E. Lipoma En la desnutricin marasmtica, el trastorno fundamental es: A. Hepatomegalia importante

B. Presencia de edemas C. Prdida de tejido graso y muscular D. Dermatitis con despigmentacin E. Irritabilidad Paciente gestante de 19 aos, con antecedente de sndrome convulsivo, con embarazo de 38 semanas, PA de 120/80 mmHg, proteinuria de 1 g/24 horas, edema +++ Cul es el diagnstico? A. Eclampsia B. Preeclampsia leve y epilepsia C. Preeclampsia severa D. Epilepsia y preeclampsia severa E. Epilepsia Los trastornos hipocondracos tienen las siguientes caractersticas, EXCEPTO: A. Muestran preocupacin exagerada por su salud B. Individuo ansioso C. Generalmente adultos jvenes D. Forman parte de las neurosis E. Personalidad paranoide Cul NO es un signo de sndrome compartimental? A. Crepitacin sea B. Prdida de sensibilidad C. Dolor post estiramiento pasivo de los msculos D. Ausencia de pulsos distales E. Equimosis gigante proximal El reflujo gastroesofgico crnico puede producir: A. Candidiasis esofgica B. Hernia hiatal C. Infeccin por Helicobacter pylori de la mucosa gstrica D. Divertculos esofgicos E. Esfago de Barrett La proporcin de incidencia en la poblacin que puede ser atribuido al factor de riesgo, se calcula con el: A. Riesgo relativo B. Riesgo atribuible poblacional porcentual C. Riesgo atribuible D. Fraccin etiolgica del riesgo E. Riesgo atribuible poblacional El cncer gstrico se localiza frecuentemente en el: A. Ploro B. Cuerpo C. Cardias D. Antro E. Fondo La conducta a seguir en una paciente gestante del primer trimestre con DIU es: A. Prohibir las relaciones sexuales B. Realizar ecografa C. Continuar embarazo D. Aborto teraputico E. Retirar DIU La induccin del trabajo de parto est contraindicada en: A. Miomectoma anterior B. Embarazo prolongado C. Ruptura prematura de membrana D. Preeclampsia E. Distocia de contraccin La relacin correcta entre los grmenes que producen neumona y la va de infeccin es: A. Legionella neumophila / hematgena B. Micoplasma pneumoniae / microaspiracin C. Streptococcus pneumoniae / inhalacin D. Hemophilus influenza / inhalacin E. SARS (Coronavirus) / inhalacin

Cul es la patologa que produce, con ms frecuencia, hipoacusia conductiva en nios?: A. Trauma acstico B. Presbiacusia C. Enfermedad de Meniere D. Otitis media aguda E. Neurinoma del acstico El enfisema subcutneo periorbitario es un signo que se presenta con mayor frecuencia en la fractura orbitaria que compromete: A. Pared medial B. Techo C. Pared lateral D. Piso E. Vrtice Cul es la arteria que permite diferenciar una hernia inguinal directa de la indirecta?: A. Femoral B. Epigstrica inferior C. Folicular D. Deferente E. Del epiddimo Caracteriza a la adolescencia temprana: A. El inters sexual pasa a un segundo plano B. La mayora alcanza el pensamiento formal C. Incremento de la autoestima D. Mayor acercamiento emocional con los padres E. Predominio de pertenencia al grupo En el tratamiento de la hiponatremia en un paciente con insuficiencia renal aguda oligrica, la medida teraputica ms apropiada es: A. Restriccin de agua libre B. Restriccin de sodio C. Administracin de sodio D. Uso de diurticos de asa E. Uso de resina de intercambio inico La mejor prueba para inferir la efectividad del control de la Diabetes Mellitus es el dosaje de: A. Glucosa en orina B. Glicemia post prandial C. Glicemia en ayunas D. Curva de tolerancia a la glucosa E. Hemoglobina glicosilada Paciente de 5 aos, ingresa a Emergencia con antecedente de ingesta de sustancia desconocida. Presenta diarrea, vmitos, bradicardia, miosis. fasciculaciones musculares, sialorrea. El sndrome txico ms probable es: A. Colinrgico B. Anticolinrgico C. Simpaticomimtico D. Extrapiramidal E. Narctico Si un nio de 8 aos hace shock anafilctico por penicilina, el medicamento de eleccin es: A. Epinefrina B. Clorfeniramina C. Dexametasona D. Dopamina E. Suero fisiolgico En un paciente adulto con hernia inguinal la reparacin quirrgica de menor recurrencia es: A. Bassini B. Lichtenstein C. Mc. Vay D. Halsted E. Ligadura alta del saco herniario Las necesidades de insulina en un paciente diabtico disminuyen con:

A. Ciruga B. Infecciones C. Stress emocional D. Obesidad E. Ejercicio En la litiasis renal por cido rico asociada a gota primaria, el frmaco a emplearse reducir la uricemia es: A. Indometacina B. Colchicina C. Furosemida D. Allopurinol E. Pirazolona En la hemobilia grave, el mtodo de diagnstico temprano ms til es: A. Arteriografa B. Gammagrafa C. Ultrasonografa D. Colangiopancreatografa retrograda E. Colangiografa transparietoheptica .Segn el Cdigo Sanitario Internacional Cul de las siguientes enfermedades est sujeta vigilancia epidemiolgica internacional?: A. Leishmaniosis B. Dengue hemorrgico C. Fiebre amarilla D. Meningitis meningoccica E. Hepatitis B La fiebre hemorrgica por virus del dengue est asociada a: A. Mayor virulencia de unas cepas que otras B. Infeccin previa por otra cepa del mismo virus C. Plaquetas en sangre normales D. Convulsiones tnico clnicas E. Secuela neurolgica . La valoracin del sinclitismo en la presentacin de vrtex, est dada por: A. Excesivo moldeamiento de la cabeza fetal B. Relacin de sutura sagital con espinas citicas C. Relacin de sutura interparietal con tuberosidades isquiticas D. Relacin de sutura sagital con promontorio y pubis E. Slo factores maternos Con relacin a la tcnica del amamantamiento, el signo que indica que el nio hace un buen agarre es: A. La nariz del beb est pegado a la areola B. La boca del bebe est semiabierta C. Se observa gran parte de la areola libre D. El mentn del bebe est separado del pecho de su madre E. El labio inferior del bebe est evertido Es caracterstica principal de la fstula intestinal baja. A. Ms del 80% de ellas cierran espontneamente B. Cuanto mayor es el gasto fistuloso el pronstico es mejor C. Causada por TBC intestinal D. Alteracin congnita E. Debe cerrarse quirrgicamente Cul de las siguientes entidades NO es causa de falla respiratoria por disfuncin neuromuscular? A. Coma mixedematoso B. Espondilitis reumatoidea C. Esclerosis lateral amiotrfica D. Difteria E. Herniacin pontina En cul de las siguientes parasitosis se observa con mayor frecuencia eosinofilia de ms del 40%? A. Giardiosis B. Fasciolosis C. Cisticercosis D. Hidatidosis E. Oxiurosis

Cul de los procedimientos proporciona un diagnstico ms preciso de Enfermedad Inflamatoria Plvica? A. Culdoscopa B. Histerosalpingografa C. Histerosonografa D. Laparoscopia E. Histeroscopa La profilaxis antibitica en ciruga est indicada en pacientes con: A. Colecistitis aguda. B. Apendicitis C. Ciruga de colon D. Vlvulo y necrosis sigmoidea E. Hernia umbilical estrangulada Cul de las condiciones precancerosas est ms relacionada al cncer gstrico y requiere vigilancia estricta?: A. lcera pptica B. Gastritis crnica atrfica C. Metaplasia intestinal D. Plipos E. Dsplasia En la pancreatitis aguda Cul de los siguientes exmenes es de valor diagnstico precoz? A. Amilasa B. Transaminasa C. Lipasa D. Frmula y recuento leucocitario E. Protena e reactiva Paciente varn que sufre una cada desde un segundo piso. Est lcido, a las 10 horas presenta prdida de conocimiento. En la radiografa de crneo se aprecia una fractura lineal a nivel parietal. El tratamiento a seguir es: A. Amplia cobertura antibitica y reposo B. Descompresin quirrgica inmediata C. Observacin y Glasgow senado D. Evolucin permanente por neurocirujano E. Manejo ambulatorio segn despierte del coma En la reaccin asmtica aguda intervienen los siguientes mediadores, EXCEPTO: A. Leucotrienos B. Oxido ntrico C. Neuropptidos D. Histamina E. Prostaglandina E2 Las lesiones malignas de la mama estn localizadas ms frecuentemente en: A. Cuadrante spero interno B. Regin retroareolar C. Cuadrante nfero externo D. Cuadrante spero externo E. Cuadrante nfero interno La tasa de incidencia de la Bartonelosis en el distrito de Caraz durante el mes de marzo de 2003 fue de 9.2 por 1000 habitantes. Cul es la interpretacin INCORRECTA de esta tasa? A. La probabilidad de que un habitante de Caraz tenga Bartonelosis es de 9.2 en 1000 B. El habitante de Caraz tiene 9.2 veces ms riesgo de enfermar por Bartonelosis C. El riesgo absoluto de la Bartonelosis en Caraz es de 9.2 en 1000 habitantes. D. El riesgo de enfermar por Bartonelosis en los habitantes de Caraz es de 9.2 en 1000 habitantes E. la probabilidad de enfermar por Bartonelosis en Caraz es do 0.0092

Se desea determinar el peso de un grupo de escolares. Cinco personas con 5 balanzas procedieron a realizar el pesaje previa calibracin antes de cada pesaje. El sesgo que se puede producir es: A. De confiabilidad B. Del observador C. De validez D. De consistencia E. Del instrumento Clnicamente las lesiones de Leishmaniosis cutnea se caracterizan porque: A. Son muy dolorosas B. No dan adenopata regional C. Curan espontneamente sin reactivacin D. Evolucionan a lceras o ndulos E. Su perodo de incubacin es menor de 7 das La complicacin ms severa de la hiperemesis gravdica es: A. Deshidratacin B. Hipotensin C. Encefalopata de Wernicke D. Hiperkalemia E. Hipernatremia Cul de los siguientes componentes pertenece a la empresa y sus relaciones externas?: A. Accionistas B. Propietario C. Funcionarios D. Empleados E. Clientes Nio nacido de parto normal hace 2 das. Desde hoy presenta edema palpebral y abundante secrecin purulenta en ambos ojos. El diagnstico ms probable es conjuntivitis neonatal por: A. Gonococo B. Clamidia C. Estreptococo D. Herpes simple E. Hemophilus La sarna se caracteriza por prurito marcado en la piel debido a: A. Infeccin agregada B. Migracin del caro C. Depsito de huevos D. Fertilizacin de las hembras E. Sensibilizacin alrgica al caro La limpieza quirrgica inicial de una fractura expuesta es muy importante y debe ser realizada: A. En el lugar del accidente. B. Una vez estabilizado el paciente C. Despus de colocar una fijacin externa D. Una semana despus del accidente E. Apenas remitan los signos de flogosis La retencin de sodio y agua por el rin en la insuficiencia cardiaca se debe a gasto cardiaco _____________, flujo sanguneo renal _________________ y fraccin de filtracin ____________. A. Aumentado / aumentado / disminuida B. Disminuido / reducido / aumentada C. Aumentado / reducido / aumentada D. Disminuido / reducido / disminuida E. Aumentado / aumentado / aumentada Uno de los siguientes hallazgos NO es de presentacin necesaria en la cetoacidosis diabtica: A. Acidosis metablica B. Glicemia mayor de 400 mg/dL C. Hiperpotasemia D. Bicarbonato menor de 15 mEq/L E. Cuerpos cetnicos en orina Paciente mujer de 15 aos, con dolor en fosa iliaca derecha de 18 horas de

evolucin y temperatura de 38.5 C desde hace 24 horas; ltima menstruacin hace 10 das. Al examen: orofaringe congestiva, abdomen blando, doloroso en fosa iliaca derecha. Cul es el diagnstico ms probable?: A. Adenitis mesentrica B. Divertculo de Meckel C. Sndrome intermenstrual D. Enfermedad inflamatoria plvica E. Diverticulitis Varn de 18 aos, fumador moderado, sin antecedente de cardiopata. Acude con dolor torcico de dos das de evolucin, sin fiebre. El ECG de ingreso muestra una elevacin difusa del segmento ST con concavidad superior. CPK-MB: 37 (n:<8). Cul es el diagnstico ms probable?: A. Osteocondritis B. Infarto agudo de miocardio C. Embolia pulmonar D. Angina inestable E. Miocarditis aguda La asociacin de leucocituria, reaccin cida de la orina y urocultivo negativo sugiere el diagnstico de: A. Glomerulonefritis aguda B. TBC renal C. Pielonefritis crnica D. Papilitis necrotizante E. Pielonefritis aguda El test de Graham permite hacer el diagnstico de: A. Trichuriosis B. Giardiosis C. Ascaridiosis D. Enterobiosis E. Balantidiosis En la amenaza de parto prematuro mayor de 34 semanas, el tratamiento para inducir la pronta madurez pulmonar fetal es el uso de: A. Corticoides B. Antibiticos C. Diurticos D. AINES E. Hormonas tiroideas En el ttanos neonatal el antibitico de eleccin es: A. Eritromicina B. Amikacina C. Penicilina D. Cefalexina E. Cloranfenicol 16. La causa ms frecuente de insuficiencia cardiaca derecha es: A. Cardiopata congnita B. Hipertensin arterial C. Miocardiopata D. Pericarditis E. Hipertensin pulmonar Paciente varn de 60 aos de edad, con hipertrofia ventricular izquierda, al examen fsico: pulso tardus, frmito carotdeo, soplo sistlico de eyeccin en regin paraesternal izquierda. El diagnstico ms probable es: A. Estenosis mitral B. Insuficiencia pulmonar C. Estenosis artica D. Insuficiencia artica E. Estenosis tricuspdea Paciente gestante de 35 semanas con sangrado vaginal escaso, dolor intenso tipo contraccin y signos de sufrimiento fetal agudo. La posibilidad diagnstica es: A. Desprendimiento prematuro de placenta B. Placenta previa

C. Inicio de trabajo de parto D. Rotura uterina E. Rotura de vasa previa La formacin de cavidades en el pulmn es una complicacin caracterstica de la neumona causada por: A. Streptococcus pneumoniae B. Legionella pneumophila C. Bacterias anaerbicas D. Micoplasma pneumoniae E. Virus de la influenza Un marcador precoz de la disminucin de la funcin heptica es: A. Trombocitopenia B. Prolongacin del tiempo parcial de tromboplastina C. Hipoproteinemia D. Anemia macroctica E. Prolongacin del tiempo de protrombina Paciente mujer de 20 aos, soltera, secretaria ejecutiva, sin antecedentes de enfermedad mental. Relata ser siempre impredecible en su conducta; cuando su progenitora le llama la atencin por llegar en estado de embriaguez; se autolesiona con cortes superficiales en el cuerpo; se torna hostil al sentirse decepcionada con su pareja, prefiere su compaa a pesar del maltrato verbal; sentimiento crnico de vaco y aburrimiento; carece de sentido de identidad consistente. Qu tipo de trastorno de personalidad es?: A. Antisocial B. Borderline C. Histrinica D. Dependiente E. Por evitacin De las siguientes unidades de un Centro de Salud: Medicina, Comit Asesor, Saneamiento Ambiental, Farmacia, Enfermera, Estadstica, Odontoestomatologa, Jefatura; son rganos de lnea: A. Estadstica, comit asesor, jefatura B. Enfermera, farmacia, odontoestomatologa C. Jefatura, medicina, saneamiento ambiental D. Medicina, saneamiento ambiental, odontoestomatologa E. Odontoestomatologa, medicina, enfermera Cul es la causa ms de frecuente de anosmia?: A. Rinitis B. Tabaquismo C. Meningioma del surco olfatorio D. Hipotiroidismo E. Hemorragia subaracnoidea En el ao 1980 se produjo un escape de gas radioactivo en una planta de fabricacin de sustancias radioactivas; la planta tena 500 obreros. La ciudad ms cercana a la planta se encontraba a 1000 Km. de distancia y tena 2000 habitantes. De 1980 al ao 2000 se diagnosticaron: 25 casos de leucemia mieloctica entre los obreros y en la ciudad ms cercana 5 casos de leucemia Cul ser la tasa de incidencia de leucemia en la poblacin expuesta?: A. 1,25 por cada 100 B. 30 por cada 1000 C. 12 por cada 1000 D. 25 por cada 100 E. 5 por cada 100 El mtodo de laboratorio de eleccin para el diagnstico del paludismo es: A. Deteccin de anticuerpos por inmunofluorescencia B. Deteccin de antgenos mediante inmunocromatografa C. Examen microscpico de muestras de sangre D. Hemocultivo E. Reaccin en cadena de la polimerasa Paciente varn de 20 aos, hace 3 das presenta dolor intenso en garganta, alza trmica de 40 C, trismus, halitosis, sialorrea, abombamiento del pilar anterior izquierdo con rechazo de la vula y la amgdala hacia lado contralateral. Cul es el diagnstico presuntivo?: A. Linfoma de Hodgkin

B. Absceso periamigdalino C. Celulitis periamigdalino D. Cncer de amgdala E. Amigdalitis aguda El msculo oponente del pulgar est inervado por el nervio: A. Cutneo B. Radial C. Cubital D. Humeral E. Mediano La tia del cuero cabelludo es producida por: A. Malassezia furfur B. Trichophyton rubrum C. Epidermophyton D. Microsporum cannis E. Microsporum tonsurans En el diagnstico de ansiedad y pnico, cul de los siguientes sntomas o signos pertenece a la tensin motora?: A. Temblores B. Taquicardia C. Palpitaciones D. Desvanecimiento E. Miccin frecuente Cul es la caracterstica principal del ectima?: A. Es un proceso que toma tejido conjuntivo laxo B. Se observan ampollas flcidas y transparentes C. Es cuadro infeccioso que abarca hasta la dermis D. Inicialmente aparece una pseudomembrana blanquecina E. Puede acompaar a enfermedades sistmicas Cul de las siguientes pruebas es la ms segura para diferenciar una anemia ferropnica de la anemia por enfermedad crnica?: A. Nivel de ferritina B. Estudio de hierro en mdula sea C. Volumen corpuscular medio D. Lmina perifrica E. Concentracin media de hemoglobina corpuscular Se evala una prueba diagnstica para enfermedad de Alzheimer, los resultados son: PRUEBA ALZHEIMER DIAGNOSTICA SI NO TOTAL + 400 200 600 - 100 300 400 500 500 1000 El valor predictivo positivo de la prueba es: A. 60% B. 75% C. 80% D. 66% E. 40% Cul de las siguientes alteraciones NO interviene en la gnesis del hiperparatiroidismo secundario de la insuficiencia renal crnica? A. Acidosis metablica B. Hipocalcemia C. Dficit de Vitamina D3 activa D. Resistencia esqueltica a la PTH E. Hipofosfatemia Los miomas uterinos submucosos son tumoraciones benignas de tejido muscular, cuyo signo ms frecuente es: A. Sangrado genital B. Aumento del tamao uterino C. Dolor plvico D. Leucorrea E. lcera cervical Paciente primigesta Rh negativa no sensibilizada con esposo Rh positivo,

recin nacido Rh positivo. La conducta a seguir es: A. Vacunarla a la semana del parto B. No requiere vacunacin hasta el prximo parto C. Vacunarla dentro de las 72 horas postparto D. Vacunarlas en cualquier momento E. Vacunar de acuerdo a la Reaccin de Coombs Dentro de los sntomas que se presentan en la migraa NO se considera: A. Localizacin unilateral B. Cefalea pulstil C. Aura asociada D. Otalgia E. Alivio de los sntomas al dormir Cul es el primer hallazgo en la obstruccin intestinal media?: A. Distensin abdominal B. Dolor clico C. Vmitos D. Ictericia E. Dificultad para eliminar flatos En un paciente que presenta oftalmoplejia, ptosis palpebral y midriasis, se sospecha compromiso de: A. Hendidura esfenoidal superior B. Agujero ptico C. Hendidura infraorbitaria D. Agujero etmoidal E. Agujero zigomtico La puntuacin de APGAR NO evala: A. Esfuerzo respiratorio B. Coloracin de la piel C. Frecuencia cardaca D. Irritabilidad refleja E. Frecuencia respiratoria El aumento del estridor en el recin nacido cuando est en decbito supino sugiere la existencia de: A. Sndrome obstructivo bronquial B. Laringotraqueitis C. Cuerpo extrao D. Laringomalacia E. Fstula traqueo esofgica Fisiopatolgicamente en el Asma Bronquial encontramos: A. Disminucin de la resistencia de las vas respiratorias B. Aumento de la insuflacin pulmonar y del trax C. Aumento en el volumen espiratorio forzado' D. Aumento del flujo areo espiratorio E. Aumento del dimetro de las vas respiratorias Cul de los siguientes es signo de hipertiroidismo?: A. Engrosamiento o ronquera de la voz B. Bradicardia C. Piel seca y gruesa D. Tumefaccin de manos, cara y extremidades E. Prdida de peso con aumento de apetito El contenido de una hernia de Littre es: A. Divertculo de Meckel B. Vejiga C. Ileon terminal D. Colon sigmoides E. Apndice cecal En el tratamiento de la bronquiolitis no complicada est indicado prioritariamente: A. Antihistamnicos B. Antibiticos C. Oxgeno hmedo D. Expectorantes

E. Antitusgenos Paciente mujer de 36 aos, con amenorrea de 6 meses; se plantea el diagnstico de menopausia precoz si se encuentra el nivel hormonal de: A. Progesterona entre 2 y 10 ng/mL B. LH entre 20 y 25 mUI/mL C. Estradiol entre 70 y 150 pg/mL D. FSH de 40 o ms mUI/mL E. Dehidroepiandrosterona entre 0,4 y 0,6 ng/mL La complicacin fetal ms frecuente en un embarazo mltiple es: A. Sepsis B. Bajo peso C. Transfusin de gemelo a gemelo D. Malformaciones congnitas E. Prematuridad La dosis peditrica de 15 mg/Kg/da de metronidazol oral en el tratamiento de la giardiosis, se administra durante _____ das: A. 8 B. 5 C. 10 D. 15 E. 12 Paciente mujer de 18 aos, llega a Emergencia por presentar: palpitaciones, ansiedad y sudoracin. Al examen: palidez, pulso: 185 por minuto regular, filiforme, hipotensin arterial. ECG: taquicardia: 185 por minuto, complejos QRS estrechos, no arritmia. Maniobras vagales negativas. El diagnstico ms probable es: A. Fibrilacin auricular B. Taquicardia ventricular C. Flutter auricular D. Taquicardia supraventricular paroxstica E. Taquicardia Sinusal Paciente de 6 meses, con tos seca exigente. Disfona precedida de secrecin nasal acuosa. Al examen: estridor inspiratorio al llanto, FR: 40 por minuto. El diagnstico ms probable es: A. Laringotraqueitis B. Epiglotitis C. Aspiracin de cuerpo extrao D. Absceso periamigdaliano E. Asma bronquial Recin nacido a las 40 semanas, cuyo peso al nacer se encuentra debajo del percentil 5 de la curva de crecimiento intrauterino de la relacin peso/edad gestacional. Cmo lo clasificara segn el percentil?: A. Adecuado para edad gestacional B. Bajo peso C. Pequeo para edad gestacional D. Extremadamente bajo peso E. Muy bajo peso En el tratamiento del asma infantil, el uso de corticoides por va inhalatoria est indicado en: A. Asma intermitente B. Asma persistente C. Asma intermitente con atelectasia D. Crisis asmtica E. Estado asmtico En la enterocolitis pseudomembranosa, el hecho ms significativo a considerar para el diagnstico es: A. Presencia de moco en las deposiciones B. Presencia de sangre en las deposiciones C. Distensin abdominal D. Antecedente de uso previo de antibiticos E. Presencia de diarrea sin moco ni sangre La lesin elemental clsica de la urticaria es: A. Habn B. Comedn

C. Ampolla D. Vescula E. Mcula En el estudio diagnstico del sndrome doloroso abdominal, cul de los siguientes actos mdicos estn contraindicados?: A. Exmenes radiolgicos y ecogrficos no invasivos B. Paracentesis y lavado peritoneal C. Analgsicos y antibiticos D. TAC y Ultrasonografa E. Exmenes videoendoscpicos En la siguiente tabla tetracrica, el clculo de la sensibilidad de una prueba diagnstica se hace mediante la frmula: PRUEBA ENFERMEDAD DIAGNSTICA SI NO +ab -cd A. b / (a + b) B. a / (a + c) C. c / (c + d) D. c / (a + c) E. d / (b + d) Cul de las siguientes manifestaciones es patognomnica del sarampin? A. Ganglios suboccipitales dolorosos B. Fiebre alta C. Eritema mculo papular generalizado D. Tos crupal E. Manchas de Koplik Las razones para recomendar el condn masculino son las siguientes, EXCEPTO: A. Es econmico y no necesita prescripcin B. Promiscuidad C. Proteccin contra infecciones de transmisin sexual D. No necesita vigilancia mdica especial E. Tiene una alta tasa de eficacia como mtodo anticonceptivo La efectividad del tratamiento de la cirrosis heptica post hepatitis C se evala mediante: A. Perfil heptico B. Ecografa C. Historia clnica D. Biopsia heptica E. Marcadores serolgicos A los cuntos meses de edad se recomienda colocar la vacuna contra la hepatitis A? A. 9 B. 12 C. 6 D. 4 E. 2 En la epistaxis posterior, el taponamiento nasal con efecto teraputico debe ser colocado en: A. Coana B. Cavum farngeo C. Nasofaringe D. Vestbulo nasal E. Vlvula turbinoseptal Paciente de 10 aos de edad, acude a Emergencia por presentar dolor de odo leve a moderado, hipoacusia. Antecedente de haber realizado natacin en das pasados y frecuente limpieza de odo con hisopo. El diagnstico ms probable es: A. Miringitis bullosa B. Otitis externa C. Otomicosis D. Tapn de cerumen E. Otitis media

Paciente con diagnstico de traumatismo encefalocraneano, fractura de base de crneo. Al examen: alerta, parcialmente desorientado, con movimientos oculares conjugados conservados y midriasis paraltica del ojo derecho. El par craneal lesionado es: A. IV B. III C. II D. V E. VI Las siguientes enfermedades son endmicas en la selva peruana, EXCEPTO: A. Malaria. B. Hepatitis viral tipo B C. Leishmaniosis D. Fiebre amarilla E. Bartonelosis Un nio de 18 meses de edad llega a Emergencia despus de una convulsin tnico clnica que dur menos de 15 minutos. Presenta fiebre desde las ltimas 24 horas. Su primera decisin es: A. Administrar diazepam B. Buscar la causa de la fiebre C. Realizar puncin lumbar D. Administrar metamizol E. Iniciar una va endovenosa En el Modelo de Atencin Integral de Salud (MAIS), los cuidados esenciales responden a ___________ y los programas de salud pblica responden a _____________: A. Necesidades de salud / riesgos y daos B. Ciclos de vida / necesidades de salud C. Riesgos y dao / necesidades de salud D. Necesidades de salud / prioridades nacionales E. Ciclos de vida / riesgos y daos Nio preescolar presenta: tos exigente de inicio brusco y prdida leve de conciencia. Al examen fsico: hipersonoridad en un hemitrax. Rx: enfisema pulmonar unilateral. Debe sospecharse: A. Epiglotitis B. Sndrome de escape de aire C. Cuerpo extrao D. Neumonitis intersticial E. Crisis asmtica La anemia normoctica normocrmica se observa en: A. Insuficiencia renal crnica B. Trastornos del metabolismo del hierro C. Trastornos del metabolismo de vitamina B12 D. Deficiencia del cido flico E. Defectos en la membrana del eritrocito Cul de las siguientes definiciones corresponde a Insuficiencia Respiratoria Aguda?: A. Incapacidad para proveer oxgeno B. Incapacidad pulmonar para satisfacer las demandas metablicas del organismo C. Incapacidad pulmonar para sostener el proceso aerbico D. Incapacidad para pasaje de oxgeno a la sangre E. Incapacidad para el intercambio gaseoso. En un nio con abdomen agudo, la presencia de _______________ indica mal pronstico; A. Niveles hidroareos B. Vmito fecaloide C. Timpanismo D. Heces sanguinolentas E. Abdomen muy doloroso En una radiografa simple de abdomen, cul de las siguientes imgenes es caracterstica del vlvulo de sigmoides?; A. Presencia de niveles hidroareos B. Distensin de la zona ileocecal

C. Gran distensin del asa sigmoidea D. Opacidad difusa del abdomen E. Velamiento del psoas Al comparar dos medias muestrales el rechazo de una hiptesis nula, que es verdadera, nos lleva a cometer un error conocido como ______________: A. Beta o Tipo II B. Aleatorio C. Estndar D. Alfa o tipo I E. Sistemtico En pacientes hipertensos, en cul de las siguientes situaciones se considera un tratamiento INADECUADO?: A. Con compromiso coronario, tratado con diltiazen B. Con taquicardia, tratado con atenolol C. Con insuficiencia renal crnica, avanzada, tratada con hidroclorotiazida D. Diabtico, tratado con captopril E. Con tos persistente, tratado con losartan En la amebiosis invasora intestinal, la dosis de metronidazol es: A. 200 mg/Kg/da B. 60 a 70 mg/Kg/da C. 100 mg/Kg/da D. 150 mg/Kg/da E. 30 a 50 mg/Kg/da En el ao 2003 se reportaron 80 casos nuevos de dengue hemorrgico en una poblacin de 25,000 habitantes, de los cuales 5 fallecieron. La incidencia y la letalidad de la enfermedad fueron: A. Incidencia 32 x 1000 hab Letalidad 6,25 x 1000 hab. B. Incidencia 3,2 x 1000 hab Letalidad 6,25 x 100 hab. C. Incidencia 3,2 x 10000 hab Letalidad 2 x 100 hab. D. Incidencia 3,2 x 100 hab Letalidad 2 x 1000 hab. E. Incidencia 0,32 x 1000 hab Letalidad 20 x 10000 hab. El diagnstico de desnutricin aguda se hace en base a la relacin: A. Talla /peso B. Peso / edad C. Talla / edad D. Peso / talla E. Peso / talla 2 La ___________ NO se relaciona directamente con el alcoholismo: A. Deficiencia de fierro B. Demencia C. Cardiomiopata D. Hepatitis E. Polineuropata perifrica En un paciente que desde muy temprana edad presenta sangrado excesivo o anormal luego de traumatismos, leve invalidez por artropata de rodillas y dosaje de factor VIII de 3%, se diagnostica: A. Enfermedad de Von Willebrand B. Hemofilia B C. Hemofilia A D. Prpura Trombocitopnica E. Tromboastenia Una complicacin frecuente de la vacuna BCG es: A. Placa verrucosa exudativa B. Absceso bacteriano secundario C. Tuberclides D. Linfadenitis regional C. Chancro tuberculoso 63. La asociacin apropiada de antibiticos para tratar la sepsis en un recin nacido es: A. Gentamicina + Cefalotina B. Amikacina + Penicilina C. Cloranfenicol + Ampicilina D. Vancomicina + Amoxicilina . E. Amikacina + Ampicilina

En relacin al crecimiento y desarrollo de un nio de 2 a 4 meses, se puede observar: A. Destreza en la comunicacin no verbal B. Aparicin de expresiones faciales claras C. Afn de llevar todo a la boca D. Autoalimentacin con los dedos E. Pronunciacin con slabas mltiples El concepto de eficiencia establece una relacin entre: A. Organizacin / recursos B. Calidad / recursos C. Logro de objetivos / recursos D. Actividades / servicios E. Logro de objetivos / metas Se define Retardo del Crecimiento Intrauterino (RCIU) como un peso al nacer por debajo del percentil: A. 10 B. 90 C. 75 D. 50 E. 25 La causa ms importante de aborto antes de las 12 semanas de gestacin es: A. Cervicitis crnica B. Infecciones sistmicas C. Infecciones vaginales D. Anomalas placentarias E. Anomalas cromosmicas Se produce enfermedad de Addison por destruccin anatmica de la glndula en: A. Metirapona B. Hiperplasia suprarrenal congnita C. Anticuerpos bloqueantes de ACTH D. Enfermedad tuberculosa E. Administracin de esteroides Caracteriza a la metaplasia escamosa cervical: A. Cambio de un epitelio cilndrico a escamoso B. Proceso muy activo en la premenopausia C. Proceso maligno D. Ocurre en la exocrvix E. Se asocia a infecciones La bacteriuria asintomtica es una entidad que slo debe tratarse en pacientes: A. Ancianos B. Gestantes C. Menopusicas D. Esplenectomizados E. Cirrticos Paciente mujer de 45 aos con Papanicolaou positivo a lesin intraepitelial alto grado. Cul es la conducta a seguir?: A. Biopsia dirigida por colposcopa B. Conizacin C. Crioterapia D. Histerectoma E. Repetir el Papanicolaou Cul de las siguientes actividades representa un riesgo laboral para contraer la brucelosis?: A. Recoleccin de algodn B. Avicultura C. Trabajo en un camal D. Trabajo en minas E. Fabricacin de cestas La manifestacin clnica ms frecuente del absceso cerebral es: A. Parlisis ocular B. Fiebre C. Letargo

D. Hemiparesia E. Cefalea Un lquido sinovial de aspecto transparente, color amarillo plido, con 0-200 leucocitos por mm3 y clulas polimorfonucleares menor del 10% es caracterstico de: A. Artritis gotosa B. Artritis pigena C. Artritis lpica D. Articulacin normal E. Artritis reumatoidea Qu porcentaje del rea bajo la curva normal estndar se localiza entre + 2 desviaciones estndares de la media?: A. 58,6 B. 95,4 C. 68,3 D. 90,4 E. 99,9 Ante una paciente primigesta de 35 aos, 42 semanas de embarazo, crvix desfavorable y oligoamnios por ecografa, la conducta a seguir es: A. Cesrea B. Induccin de trabajo de parto C. Observacin y monitoreo materno fetal D. Parto eutcico E. Repetir la ecografa en 48 horas La causa ms frecuente de pancreatitis aguda es: A. Obstruccin duodenal B. Consumo de drogas C. Enfermedad biliar D. Infeccin entrica E. Veneno de escorpin El acto mdico ms importante en el tratamiento inicial de una fractura de tibia es: A. Evitar el shock hipovolmico B. Inmovilizacin del miembro afectado C. Administrar antibiticos D. Tranquilizar al paciente E. Evitar el sndrome compartamental De acuerdo al Manual del MINSA, el medicamento de eleccin en la bartonelosis aguda, es: A. Ciprofloxacino B. Penicilina procanica C. Estreptomicina D. Tetraciclinas E. Cloranfenicol Se desea estimar la prevalencia de una enfermedad que se sospecha tiene un comportamiento estacional. Para obtener una muestra representativa se debe optar por el muestreo: A. Estratificado B. Aleatorio simple con restitucin C. Aleatorio simple sin restitucin D. Sistemtico E. Por conglomerado En el diagnstico de polimiositis, la enzima que se incrementa significativamente es: A. Transaminasa oxalactica B. Transaminasa pirvica C. Deshidrogenasa lctica D. Creatinfosfoquinasa E. Aldolasa La primera manifestacin del climaterio es: A. Sensacin de bochorno B. Trastornos en el ciclo menstrual C. Cambios atrficos en los genitales externos D. Cambios atrficos en las mamas

E. Disminucin de la estatura Cul es la caracterstica ms importante de la lesin para el pronstico de un paciente con melanoma maligno?: A. Grosor B. Dimetro C. Borde D. Color E. Vascularidad

Vous aimerez peut-être aussi